med surg exam 3 Nclex

Lakukan tugas rumah & ujian kamu dengan baik sekarang menggunakan Quizwiz!

A nurse is assigned to care for a client with nephrotic syndrome. The nurse assesses which important parameter on a daily basis? a) weight b) albumin levels c) activity tolerance d) blood urea nitrogen (BUN) level

weight

"The nurse is caring for a patient following an appendectomy. The patient takes a deep breath, coughs, and then winces in pain. Which of the following statements, if made by the nurse to the patient, is BEST? "A.) "Take three deep breaths, hold your incision, and then cough." B.) "That was good. Do that again and soon it won't hurt as much." C.) "It won't hurt as much if you hold your incision when you cough." D.) "Take another deep breath, hold it, and then cough deeply."

"(1) correct-most effective way of deep breathing and coughing, dilates airway and expands lung surface area (2) should splint incision before coughing to reduce discomfort and increase efficiency (3) partial answer, should take three deep breaths before coughing (4) implies coughing routine is adequate, incision needs to be splinted"

"Which of the following diets would be most appropriate for the client with ulcerative colitis? 1. High-calorie, low-protein. 2. High-protein, low-residue. 3. Low-fat, high-fiber. 4. Low-sodium, high-carbohydrate."

"2. High protein, low residue Clients with ulcerative colitis should follow a well-balanced high-protein, high-calorie, low-residue diet, avoiding such high-residue foods as whole-wheat grains, nuts, and raw fruits and vegetables. Clients with ulcerative colitis need more protein for tissue healing and should avoid excess roughage. There is no need for clients with ulcerative colitis to follow low-sodium diets."

"The nurse explains to the patient with gastoesophageal reflux disease that this disorder "a. result in acid erosion and ulceration of the esophagus caused by frequent vomiting. b. will require surgical wrapping or repair of the pyloric sphincter to control the symptoms. c. is the protrusion of a portion of the stomach into the esophagus through an opening in the diaphragm. D. often involves relaxation of the lower esophageal sphincter, allowing stomach contents to back up into the esophagus

"4. Correct answer: d Rationale: Gastroesophageal reflux disease (GERD) results when the defenses of the esophagus are overwhelmed by the reflux of acidic gastric contents into the lower esophagus. An incompetent lower esophageal sphincter (LES) is a common cause of gastric reflux."

The family of a patient newly diagnosed with hepatitis A asks the nurse what they can do to prevent becoming ill themselves. Which response by the nurse is most appropriate? "The hepatitis vaccine will provide immunity from this exposure and future exposures." "I am afraid there is nothing you can do since the patient was infectious before admission." "You will need to be tested first to make sure you don't have the virus before we can treat you." "An injection of immunoglobulin will need to be given to prevent or minimize the effects from this exposure."

"An injection of immunoglobulin will need to be given to prevent or minimize the effects from this exposure." Immunoglobulin provides temporary (1-2 months) passive immunity and is effective for preventing hepatitis A if given within 2 weeks after exposure. It may not prevent infection in all persons, but it will at least modify the illness to a subclinical infection. The hepatitis vaccine is only used for preexposure prophylaxis.

"A client with appendicitis is experiencing excruciating abdominal pain. An abdominal X-ray film reveals intraperitoneal air. The nurse should prepare the client for: a) colonoscopy. b) surgery. c) nasogastric (NG) tube insertion. d) barium enema."

"B) Surgery The client should be prepared for surgery because his signs and symptoms indicate bowel perforation. Appendicitis is the most common cause of bowel perforation in the United States. Because perforation can lead to peritonitis and sepsis, surgery wouldn't be delayed to perform other interventions, such as colonoscopy, NG tube insertion, or a barium enema. These procedures aren't necessary at this point."

"Which goal for the client's care should take priority during the first days of hospitalization for an exacerbation of ulcerative colitis? source: "A:promoting self care and independence B:managing diarrhea C:maintaining adequate nutrition D:promoting rest and comfort"

"B. managing diarrhea Diarrhea is the primary symptom in an exacerbation of ulcerative colitis, and decreasing the frequency of stools is the first goal of treatment. The other goals are ongoing and will be best achieved by haulting the exacerbation. The client may recieve antidiarrheal agents, antispasmodic agents, bulk hydrophilic agents, or anti-inflammatory drugs."

The nurse provides discharge instructions for a 64-year-old woman with ascites and peripheral edema related to cirrhosis. Which statement, if made by the patient, indicates teaching was effective? "It is safe to take acetaminophen up to four times a day for pain." "Lactulose (Cephulac) should be taken every day to prevent constipation." "Herbs and other spices should be used to season my foods instead of salt." "I will eat foods high in potassium while taking spironolactone (Aldactone)."

"Herbs and other spices should be used to season my foods instead of salt." A low-sodium diet is indicated for the patient with ascites and edema related to cirrhosis. Table salt is a well-known source of sodium and should be avoided. Alternatives to salt to season foods include the use of seasonings such as garlic, parsley, onion, lemon juice, and spices. Pain medications such as acetaminophen, aspirin, and ibuprofen should be avoided as these medications may be toxic to the liver. The patient should avoid potentially hepatotoxic over-the-counter drugs (e.g., acetaminophen) because the diseased liver is unable to metabolize these drugs. Spironolactone is a potassium-sparing diuretic. Lactulose results in the acidification of feces in bowel and trapping of ammonia, causing its elimination in feces.

When teaching the patient with acute hepatitis C (HCV), the patient demonstrates understanding when the patient makes which statement? "I will use care when kissing my wife to prevent giving it to her." "I will need to take adofevir (Hepsera) to prevent chronic HCV." "Now that I have had HCV, I will have immunity and not get it again." "I will need to be checked for chronic HCV and other liver problems."

"I will need to be checked for chronic HCV and other liver problems." The majority of patients who acquire HCV usually develop chronic infection, which may lead to cirrhosis or liver cancer. HCV is not transmitted via saliva, but percutaneously and via high-risk sexual activity exposure. The treatment for acute viral hepatitis focuses on resting the body and adequate nutrition for liver regeneration. Adofevir (Hepsera) is taken for severe hepatitis B (HBV) with liver failure. Chronic HCV is treated with pegylated interferon with ribavirin. Immunity with HCV does not occur as it does with HAV and HBV, so the patient may be reinfected with another type of HCV.

The nurse instructs a 50-year-old woman about cholestyramine to reduce pruritis caused by gallbladder disease. Which statement by the patient to the nurse indicates she understands the instructions? "This medication will help me digest fats and fat-soluble vitamins." "I will apply the medicated lotion sparingly to the areas where I itch." "The medication is a powder and needs to be mixed with milk or juice." "I should take this medication on an empty stomach at the same time each day."

"The medication is a powder and needs to be mixed with milk or juice." For treatment of pruritus, cholestyramine may provide relief. This is a resin that binds bile salts in the intestine, increasing their excretion in the feces. Cholestyramine is in powder form and should be mixed with milk or juice before oral administration.

A client is admitted with irritable bowel syndrome. The nurse would anticipate the client's history to reflect which of the following? 1. Pattern of alternating diarrhea and constipation. 2. Chronic diarrhea stools occurring 10-12 times per day. 3. Diarrhea and vomiting with severe abdominal distention. 4. Bloody stools with increased cramping after eating.

(1) correct-condition is often called spastic bowel disease; no inflammation is present (2) refers to inflammatory bowel disease such as ulcerative colitis or Crohn's disease (3) refers to inflammatory bowel disease such as ulcerative colitis or Crohn's disease (4) bloody stools do not occur 38.

When planning care for a client with ulcerative colitis who is experiencing an exacerbation of symptoms, which client care activities can the nurse appropriately delegate to an unlicensed assistant? Select all that apply. 1. Assessing the client's bowel sounds. 2. Providing skin care following bowel movements. 3. Evaluating the client's response to antidiarrheal medications. 4. Maintaining intake and output records. 5. Obtaining the client's weight.

(2,4, & 5 are CORRECT)The nurse can delegate the following basic care activities to the unlicensed assistant: providing skin care following bowel movements, maintaining intake and output records, and obtaining the client's weight. Assessing the client's bowel sounds and evaluating the client's response to medication are registered nurse activities that cannot be delegated.

the nurse caring for a client 1 day postop sigmoid resection notes a moderate amount of dark reddish brown drainage on the midline abdominal incision. which intervention should the nurse implement first? 1. mark the drainage on the dressing with the time and date 2. change the dressing immediately using sterile technique 3. notify the hcp immediately 4. reinforce the dressing with a sterile gauze pad

1 (the nurse should mark the drainage on the dressing to determine if active bleeding is occuring, because dark reddish brown drainage indicates old blood. this allows the nurse to assess what is actually happening)

the client has dark, watery, and shiny appearing stool. which intervention should the nurse implement first? 1. check for fecal impaction 2. encourage the client to drink fluids 3. check the chart for sodium and potassium levels 4. apply a protective barrier cream to the perianal area

1 (this is a symptom of diarrhea moving around an impaction higher up in the colon. the nurse should assess for an impaction when observing this finding.)

What instructions should be given to a patient with a cast to prevent edema and skin breakdown? Select all that apply. 1 Elevate the affected limb above heart level during the first 48 hours. 2 Apply ice on the fracture site during the first 24 hours. 3 Cover the cast with plastic for prolonged periods. 4 Remove the padding of the cast after going home.

1 Elevate the affected limb above heart level during the first 48 hours. 2 Apply ice on the fracture site during the first 24 hours. 5 Exercise joints above and below the cast. Regardless of the type of cast material, a cast can interfere with circulation and nerve function if edema occurs after the application of the cast. Therefore, it is important to teach the patient and caregivers about measures to prevent edema. Elevating the limb above heart level and applying ice in the initial phase (24 to 48 hours) help in preventing edema. The patient should also be encouraged to exercise the joints above and below the cast. Pulling out the cast padding, inserting foreign objects into the cast to scratch an itch, and covering the cast with plastic may predispose patients to skin breakdown and infections. Therefore they should be advised to abstain from doing such things.

When treating a patient with compartment syndrome, what measures should the nurse consider to be contraindicated? Select all that apply. 1 Elevation of the limb above heart level. 2 Bivalving of the bandage. 3 Application of cold compresses. 4 Reduction in traction weight. 5 Bandage removal.

1 Elevation of the limb above heart level. 3 Application of cold compresses. Elevation of the extremity may lower venous pressure and slow arterial perfusion. Therefore the extremity should not be elevated above heart level in case of compartment syndrome. Similarly, the application of cold compresses may result in vasoconstriction and exacerbate compartment syndrome. It may also be necessary to remove or loosen the bandage and split the cast in half (bivalving). A reduction in traction weight may also decrease external circumferential pressures.

A nurse is caring for a patient with a fractured femur. The health care provider finds that the patient has fat embolism syndrome. What treatment (or treatments) of fat embolism syndrome should the nurse anticipate for this patient? Select all that apply. 1 Fluid resuscitation 2 Avoidance of coughing 3 Correction of acidosis 4 Fracture immobilization 5 Frequent change in positions

1 Fluid resuscitation 3 Correction of acidosis 4 Fracture immobilization The treatment of fat embolism syndrome is directed toward the management of symptoms. This includes fluid resuscitation to prevent hypovolemic shock, correction of acidosis, and fracture immobilization. The patient should be encouraged to cough and perform deep breathing. The patient should be repositioned as little as possible to prevent dislodgment of fat droplets into the general circulation.

After a motor vehicle collision, a 30-year-old man has a dislocated right hip joint, and his right thigh bone is fractured into three pieces, with one piece of the bone exposed. What type of fracture should the nurse document? Select all that apply. 1 Open 2 Comminuted 3 Closed 4 Displaced 5 Greenstick

1 Open 2 Comminuted 4 Displaced In this case, the bone is exposed, and therefore it is an open fracture. Comminuted fractures have two or more fragments of bones. Fractures can be classified as displaced or nondisplaced. In a displaced fracture, the two ends of the broken bone are separated from one another and are out of their normal positions. This fracture is not a closed one, because the fractured bone is exposed through soft tissue injury. Greenstick fracture is a type of fracture in which the periosteum is intact across the fracture and the bone is still in alignment.

the nurse is planning the care of a client who has had an abdominal perineal resection for cancer of the colon, which interventions should the nurse implement? select all that apply 1. provide meticulous skin care to stoma 2. assess the flank incision 3. maintain the indwelling cath 4. irrigate the JP drains every shift 5. position the client semirecumbent

1,3,5 (colostomy stomas are opening through the abdominal wall into the colon, through which feces exit the body. feces can be irritating to the abd skin, so careful and thourough skin care is needed. There are midline and perineal incisions not flank incisions. because of the perineal wound, the client will have an indwelling catheter to keep urine out of the incision. JP drains are emptied every shift, but not irrigated. The client should not sit upright because this causes pressure on the perinuem)

The client is diagnosed with Crohn's disease, also known as regional enteritis. Which statement by the client would support this diagnosis? 1. "My pain goes away when I have a bowel movement" 2. "I have bright red blood in my stool all the time" 3. "I have episodes of diarrhea and constipation" 4. "My abdomen is hard and rigid and I have a fever".

1. (CORRECT) The terminal ileum is the most common site for regional enteritis and causes right lower quadrant pain that is relieved by defecation 2. Stools are liquid or semi-formed and usually do not contain blood 3. Episodes of diarrhea and constipation may be a sign/symptom of colon cancer, not Crohn'sdisease 4. A fever and hard rigid abdomen are signs/symptoms of peritonitis, a complication of Crohn's disease

The client is diagnosed with an acute exacerbation of ulcerative colitis. Which intervention should the nurse implement? 1. Provide a low-residue diet. 2. Monitor intravenous fluids. 3. Assess vital signs daily. 4. Administer antacids orally.

1. The client's bowel should be placed on rest and no foods or fluids should be introduced into the bowel. 2. (CORRECT) The client requires fluids to help prevent dehydration from diarrhea and to replace fluid lost through normal body functioning. 3. The vital signs must be taken more often than daily in a client who is having an acute exacerbation of ulcerative colitis. 4. The client will receive anti-inflammatory and antidiarrheal medications, not antacids, which are used for gastroenteritis.

the nurse is caring for clients on a surgical unit. which client should the nurse assess first? 1. the client who had an inguinal hernia repair and has not voided in 4 hours 2. the client who was admitted with abdominal pain who suddenly has no pain 3. the client 4 hours postop abdominal surgery with no bowel sounds 4. the client who is 1 day postappendectomy and is being discharged

2 (a sudden cessation of pain may indicate a ruptured appendix, which could lead to peritonitis, a life threatening complication; therefore, the nurse should assess this client first)

the client dx with ulcerative colitis has an ileostomy. which statement indicates the client needs more teaching concerning the ileostomy? 1. my stoma should be pink and moist 2. i will irrigate my ileostomy every morning 3. if i get a red bumpy, itchy rash i will call my hcp 4. i will change my pouch if it starts leaking

2 (i will irrigate my ileostomy every morning: an ileostomy will drain liquid all the time and shouldnt routinely be irrigated. a sigmoid colostomy may need daily irrigation to evacuate feces)

A patient with a fracture of the right tibia has been advised to use a cane. What instructions should the nurse give to this patient? Select all that apply. 1 Hold the cane in the right hand. 2 Hold the cane in the left hand. 3 Advance the right leg first. 4 Advance the left leg first. 5 Advance the right leg last. 6 Advance the left leg last.

2 Hold the cane in the left hand. 3 Advance the right leg first. 6 Advance the left leg last. The decision to use assistive devices is made by the health care provider depending on the needs and lifestyle of patient. Use of these devices varies. When a cane is used, the affected limb is advanced along with or after the cane, and the unaffected limb is advanced last. The cane is held in the hand opposite the affected limb. In this case, the right leg is affected, and therefore the cane should be held in the left hand, the right leg should be advanced first, and the left leg should be advanced last.

The nurse is caring for a patient with a closed ulna fracture after the application of a cast. After the patient complains of severe pain, the nurse administers an analgesic and applies an ice pack. When the patient states that the medication provided no relief, the nurse should be alert to the possible of: 1. Anxiety 2. Arterial compromise 3. Infection 4. The short time frame since the ulna is fracture.

2. Arterial compromise. Rationale: Fracture pain is usually relieved by analgesics, elevation, cold packs, and rest. Pain that is not improved from these interventions is most likely due to impaired perfusion and ischemia. The nurse should assess for vascular compromise and then notify the physician immediately. Anxiety often increases pain, but failure to relieve pain from normal interventions is suspicious of vascular compromise. Infection and is accompanied pain do not develop immediately.

The nurse is reviewing the x-ray of a patient with a fractured humerus. The x-ray shows one side of the bone is broken while the other side is bent. What type of fracture is this? 1. Compression fracture 2. Greenstick fracture 3. Impacted fracture 4. Oblique fracture

2. Greenstick fracture Rationale: A greenstick fracture is when one side of the bone is broken while the other side is only bent. These fractures most occur often in children. An impacted fracture is when a part of the bone is driven into another bone. A compression fracture is when the fractured bone is compressed by other bone tissue. An oblique fracture is when the fracture is diagonal to the bone's long axis.

The nurse is assessing a patient in a Thomas splint traction, a device used to immobilize a hip fracture. The nurse should assess for complications as indicated by: 1. Pain at the site of a fracture 2. Redness on the skin that does not blanch when pressed 3. Tight bandages overlapping the length of the leg from ankle to groin 4. Warm toes 5. Weak posterior tibial pulse

2. Redness on the skin that does not blanch when pressed 5. Weak posterior tibial pulse Rationale: Weak posterior tibial pulses are a sign of poor perfusion and arterial compromise. This could be caused by tissue pressure from the traction devise. Redness that doesn't blanch with pressure is a sign of a developing pressure ulcer, a complication possible with the pressure of the bandage with a Thomas splint. A Thomas splint applies traction by use of bandages that overlap the length of the entire leg, applying the pressure evenly with weight at the bottom. This is an expected finding. Warm toes are a sign of good perfusion. Pain at the fracture site is an expected finding.

A patient has presented with an unstable wrist fracture. What kind of cast is best for this patient? 1 Posterior splint 2 Short arm cast 3 Long arm cast 4 Sugar-tong splint

3 The long arm cast is commonly used for stable forearm or elbow fractures and unstable wrist fractures. It is similar to the short arm cast but extends to the proximal humerus, restricting motion at the wrist and the elbow. The sugar-tong posterior splint accommodates post injury swelling in the fractured extremity. A short arm cast is used for stable wrist fractures. The sugar-tong splint is typically used for acute wrist injuries or injuries that may result in significant swelling.

the nurse is discussing the therapeutic diet for the client dx with diverticulosis. which meal indicates the client understands the discharge teaching? 1. fried fish, mashed potatoes, and iced tea 2. ham sandwich, applesauce, and whole milk 3. chicken salad on whole wheat bread and water 4. lettuce, tomato, and cucumber salad and coffee.

3 (chicken salad, which has vegetables such as celery, grapes, and apples, and whole wheat bread are high in fiber, which is the therapeutic diet prescribed for clients with diverticulosis. an adequate intake of water helps prevent constipation. fried foods increase cholesterol. mashed potatoes do not have the peel, which is needed for increased fiber. applesauce does not have the peel, which is needed for increased fiber, and the option does not identify which type of bread; whole milk is high in fat

the client dx with an acute exacerbation of IBD. which priority intervention should the nurse implement first? 1. weigh the client daily and document in the clients chart 2. teach coping strategies such as dietary modifications 3. record the frequency, amount, and color of stools 4. monitor the clients oral fluid intake every shift

3 (record the frequency, amount, and color of stools: the severity of the diarrhea helps determine the need for fluid replacement. the liquid stool should be measured as part of the total output. 1. weighing the pt will be included but not the priority 2.coping strategies help develop healthy ways to deal with this chronic disease, which has remissions and exacerbations but its not the priority 4. the client will be NPO when theres an acute exacerbation to allow the bowel to rest)

the client dx with ulcerative colitis is prescribed a low residue diet. which meal selection indicates the client understands the diet teaching 1. grilled hamburger on a wheat bun and fried potatoes 2. a chicken salad sandwich and lettuce and tomato salad 3. roast pork, white rice, and plain custard 4. fried fish, whole grain pasta, and fruit salad

3 (roast pork, white rice, and plain custard: a low residue diet is low fiber diet. products made of refined flour or finely milled grains, along with roasted baked or broiled meats are recommended. Raw vegetables should be avoided because this is roughage)

the nurse is caring for clients in an outpatient clinic. which information should the nurse teach regarding the american cancer societys recommendations for the early detection of colon cancer? 1. beginning at age 60, a digital rectal exam should be done yearly 2. after reaching middle age, a yearly fecal occult blood test should be done 3. have a colonoscopy at age 50 and then once every 5 to 10 yrs 4. a flexible sigmoidoscopy should be done yearly after age 40

3 (the american cancer society recommends a colonoscopy at age 50 and every 5 to 10 yrs thereafter, and a flexible sigmoidoscopy and a barium enema every 5 yrs)

client is admitted to the medical unit with a dx of acute diverticulitis. which hcp order should the nurse question? 1. insert an ng tube 2. start an IV with D5W at 125 mL/hr 3. put client on a clear liquid diet 4. place client on bedrest with bathroom privileges

3 (the nurse should question a clear liquid diet bc the bowel must be put on total rest, which means NPO. the client will have a ng tube bc the client will be NPO, which will decompress the bowel and remove hydrochloric acid. preventing dehydration is a priority with the client who is NPO. The client is in severe pain and should be on bedrest, which will help rest the bowel.)

which of the following would be included in the routine care of an ileostomy? 1. bowel training, thus eliminating need to wear appliance 2. irrigating the stoma daily to promote continence 3. providing a skin barrier around the stoma 4. administering a stool softner daily

3 (to maintain skin integrity, the nurse would ensure that the skin is clean and dry and then apply a skin barrier and a properly fitting appliance. bowel training isnt done with ileostomies bc the contets are liquid and contain digestive enzymes. Ileostomies are not routinely irrigated. because the contents are liquid stool softeners are not needed)

A nurse is caring for a patient who has just sustained a hip fracture. Which nursing action should be performed first? 1 Administering pain medication 2 Preparing for immediate surgery 3 Immobilizing the affected extremity 4 Placing the injured extremity in traction

3 Immobilizing the affected extremity The priority of emergency management for a fractured hip is immobilizing the affected extremity, because movement could cause further damage and more extensive internal bleeding and worsen the patient's pain. Administering pain medication, preparing the patient for immediate surgery, and placing the injured extremity in traction are secondary nursing interventions that require further direction from the primary health care provider.

the client developed a paralytic ileus after abdominal surgery. which intervention should the nurse include in the plan of care? 1. administer a laxative of choice 2. encourage client to increase oral fluids 3. encourage client to take deep breaths 4. maintain a patent ng tube

4 (a paralytic ileus is the absence of peristalsis; therefore, the bowel will be unable to process any oral intake. a ng tube is inserted to decompress the bowel until surgical intervention or until bowel sounds return spontaneously)

the nurse is teaching the client dx with diverticulosis. which instruction should the nurse include in the teaching session? 1. discuss the importance of drinking 1000 mL of water daily. 2. instruct the client to exercise at least 3 times a week 3. teach the client about eating a low residue diet 4. explain the need to have daily bowel movement

4 (the client should have regular bowel movements, preferably daily. constipation may cause diverticulitis, which is a potentially life threatening complication of diverticulosis. the client should drink at least 3000 mL of water daily to help prevent constipation. the client should exercise daily to help prevent constipation, the client should eat a HIGH fiber diet to prevent constipation)

the client who has had an abdominal perineal resection is being discharged. which discharge information should the nurse teach? 1. the stoma should be a white, blue or purple color. 2. limit ambulation to prevent the pouch from coming off 3. take pain medication when the pain level is at an 8 4. empty the pouch when it is one third to one half full

4 (the pouch should be emptied when it is one third to one half full to prevent the contents from becoming too heavy for the seal to hold and to prevent leakage from occuring.)

"A client is admitted with inflammatory bowel syndrome (Crohn's disease). Which treatment measures should the nurse expect to be part of the care plan? SELECT ALL THAT APPLY! "1) Laculose therapy 2) High fiber diet 3) High protein milkshakes 4) Corticosteroid therapy 5) Antidiarrheal medications

4) Corticosteroid therapy 5) Antidiarrheal medications

A client with inflammatory bowel disease (IBD) requires an ileostomy. The nurse would instruct the client to do which of the following measures as an essential part of caring for the stoma? 1.Perform massage of the stoma three times a day. 2.Include high-fiber foods in the diet, especially nuts. 3.Limit fluid intake to prevent loose stools. 4.Cleanse the peristomal skin meticulously.

4; cleanse peristomal skin meticulously1.It is not an intervention used for ileostomies. 2.Clients should avoid the high-fiber and gas-producing foods. 3.These clients are not on fluid restriction. 4.Careful cleansing is necessary to prevent skin breakdown and skin irritation.

when developing a plan of care for a client with viral hepatitis, the nurse should incorporate nursing orders that reflect the primary treatment. emphasis will be on ensuring that the client which of the following A. adequate bed rest B. generous fluid intake C. regular antibiotic therapy D. daily intravenous electrolyte therapy

A

After providing discharge instructions to a patient following a laparoscopic cholecystectomy, the nurse recognizes that teaching has been effective when the patient states, a. "I can remove the bandages on my incisions tomorrow and take a shower." b. "I can expect some yellow-green drainage from the incision for a few days." c. "I should plan to limit my activities and not return to work for 4 to 6 weeks." d. "I will always need to maintain a low-fat diet since I no longer have a gallbladder."

A After a laparoscopic cholecystectomy, the patient will have Band-Aids in place over the incisions. Patients are discharged the same (or next) day and have few restrictions on activities of daily living. Drainage from the incisions would be abnormal, and the patient should be instructed to call the health care provider if this occurs. A low-fat diet may be recommended for a few weeks after surgery but will not be a life-long requirement.

A homeless patient with severe anorexia, fatigue, jaundice, and hepatomegaly is diagnosed with viral hepatitis and has just been admitted to the hospital. In planning care for the patient, the nurse assigns the highest priority to the patient outcome of a. maintaining adequate nutrition. b. establishing a stable home environment. c. increasing activity level. d. identifying the source of exposure to hepatitis.

A Rationale: The highest priority outcome is to maintain nutrition because adequate nutrition is needed for hepatocyte regeneration. Finding a home for the patient and identifying the source of the infection would be appropriate activities, but they do not have as high a priority as having adequate nutrition. Although the patient's activity level will be gradually increased, rest is indicated during the acute phase of hepatitis.

The nurse performs a detailed assessment of the abdomen of a patient with a possible bowel obstruction, knowing that a manifestion of an obstruction in the large intestine is (select all that apply) a ) a largely distended abdomen b) diarrhea that is loose or liquid c) persistent, colicky abdominal pain d) profuse vomiting that relieves abdominal pain.

A & C: distended abd + colicky abd painPersistent, colicky abdominal pain is seen with lower intestinal obstruction. Abdominal distention is markedly incerased in lower interestinal obstructions. Onset of a large intestine obstruction is gradual, vomiting is rare, and there is usually absolute constipation.

The nurse is caring for a woman recently diagnosed with viral hepatitis A. Which individual should the nurse refer for an immunoglobin (IG) injection? A caregiver who lives in the same household with the patient A friend who delivers meals to the patient and family each week A relative with a history of hepatitis A who visits the patient daily A child living in the home who received the hepatitis A vaccine 3 months ago

A caregiver who lives in the same household with the patient IG is recommended for persons who do not have anti-HAV antibodies and are exposed as a result of close contact with persons who have HAV or foodborne exposure. Persons who have received a dose of HAV vaccine more than 1 month previously or who have a history of HAV infection do not require IG.

When providing discharge teaching for the patient after a laparoscopic cholecystectomy, what information should the nurse include? A lower-fat diet may be better tolerated for several weeks. Correct Do not return to work or normal activities for 3 weeks. Bile-colored drainage will probably drain from the incision. Keep the bandages on and the puncture site dry until it heals.

A lower-fat diet may be better tolerated for several weeks. Correct Although the usual diet can be resumed, a low-fat diet is usually better tolerated for several weeks following surgery. Normal activities can be gradually resumed as the patient tolerates. Bile-colored drainage or pus, redness, swelling, severe pain, and fever may all indicate infection. The bandage may be removed the day after surgery, and the patient can shower.

A client's stool are light gray in color. The nurse should asses the client for which of the following? Select all that apply. a. Intolerance to fatty foods b. Fever c. Jaundice d. Respiratory distress e. Pain at McBurney's point f. Peptic ulcer disease

A, B, C: Bile is created in the liver, stored in the gallbladder, and released into the duodenum, giving stool its brown color. A bile duct obstruction can cause pale-colored stool. Respiratory distress is not a symptom. Pain at McBurney's point is associated with appendicitis. Bleeding ulcers produce black tarry stool.

In addition to urine function, the nurse recognizes that the kidneys perform numerous other functions important to the maintenance of homeostasis. Which physiologic processes are performed by the kidneys? Select all that apply. A. Production of renin B. Activation of vitamin D C. Carbohydrate metabolism D. Erythropoietin production E. Hemolysis of old red blood cells (RBCs)

A, B, D. In addition to urine formation, the kidneys release renin to maintain blood pressure, activate vitamin D to maintain calcium levels, and produce erythropoietin to stimulate RBC production. Carbohydrate metabolism and hemolysis of old RBCs are not physiologic functions that are performed by the kidneys.

A patient with a history of peptic ulcer disease has presented to the emergency department with complaints of severe abdominal pain and a rigid, boardlike abdomen, prompting the health care team to suspect a perforated ulcer. Which of the following actions should the nurse anticipate? a. Providing IV fluids and inserting a nasogastric tube; b. Administering oral bicarbonate and testing the patient's gastric pH level; c. Performing a fecal occult blood test and administering IV calcium gluconate; d. Starting parenteral nutrition and placing the patient in a high-Fowler's position;

A, IV fluids + NG tubeA perforated peptic ulcer requires IV replacement of fluid losses and continued gastric aspiration by NG tube. Nothing is given by mouth and gastric pH testing is not a priority. Calcium gluconate is not a medication directly relevant to the patient's suspected diagnosis and parenteral nutrition is not a priority in the short term

"The nurse is teaching about irritable bowel syndrome. Which of the following would be most important? "A. Reinforcing the need for a balanced diet B. Encouraging the client to drink 16 ounces of fluid with each meal C. Telling the client to eat a diet low in fiber D. Instructing the client to limit his intake of fruits and vegetables"

A, reinforce the need for balanced dietThe nurse should reinforce the need for a diet balanced in all nutrients and fiber. Foods that often cause diarrhea and bloating associated with irritable bowel syndrome include fried foods, caffeinated beverages, alcohol, and spicy foods. Therefore, answers B, C, and D are incorrect.

in a client with viral hepatitis, the nurse should closely assess for indications of which of the following abnormal lab results A. prolonged prothrombin time B. decreased blood glucose level C. elevated serum potassium level D. decreased serum calcium level

A. prothrombin time may be prolonged because of decreased absorption of vitamin K and decreased production of prothrombin by the liver

When providing discharge teaching for the patient after a laparoscopic cholecystectomy, what information should the nurse include? A. A lower-fat diet may be better tolerated for several weeks. B. Do not return to work or normal activities for 3 weeks. C. Bile-colored drainage will probably drain from the incision. D. Keep the bandages on and the puncture site dry until it heals.

A. A lower-fat diet may be better tolerated for several weeks. Although the usual diet can be resumed, a low-fat diet is usually better tolerated for several weeks following surgery. Normal activities can be gradually resumed as the patient tolerates. Bile-colored drainage or pus, redness, swelling, severe pain, and fever may all indicate infection. The bandage may be removed the day after surgery, and the patient can shower.

Which nursing diagnosis is a priority in the care of a patient with renal calculi? A. Acute pain B. Risk for constipation C. Deficient fluid volume D. Risk for powerlessness

A. Acute pain Urinary stones are associated with severe abdominal or flank pain. Deficient fluid volume is unlikely to result from urinary stones, whereas constipation is more likely to be an indirect consequence rather than a primary clinical manifestation of the problem. The presence of pain supersedes powerlessness as an immediate focus of nursing care.

Which of the following symptoms indicated diverticulosis? A. No symptoms exist B. Change in bowel habits C. Anorexia with low-grade fever D. Episodic, dull, or steady midabdominal pain

A. Diverticulosis is an asymptomatic condition. The other choices are signs and symptoms of diverticulitis.

Which of the following types of diets is implicated in the development of diverticulosis? A. Low-fiber diet B. High-fiber diet C. High-protein diet D. Low-carbohydrate diet

A. Low-fiber diets have been implicated in the development of diverticula because these diets decrease the bulk in the stool and predispose the person to the development of constipation. A high-fiber diet is recommended to help prevent diverticulosis. A high-protein or low-carbohydrate diet has no effect on the development of diverticulosis.

Fistulas are most common with which of the following bowel disorders? A. Crohn's disease B. Diverticulitis C. Diverticulosis D. Ulcerative colitis

A. The lesions of Crohn's disease are transmural; that is, they involve all thickness of the bowel. These lesions may perforate the bowel wall, forming fistulas with adjacent structures. Fistulas don't develop in diverticulitis or diverticulosis. The ulcers that occur in the submucosal and mucosal layers of the intestine in ulcerative colitis usually don't progress to fistula formation as in Crohn's disease.

The patient with cirrhosis has an increased abdominal girth from ascites. The nurse should know that this fluid gathers in the abdomen for which reasons (select all that apply)? A. There is decreased colloid oncotic pressure from the liver's inability to synthesize albumin. B. Hyperaldosteronism related to damaged hepatocytes increases sodium and fluid retention. C. Portal hypertension pushes proteins from the blood vessels, causing leaking into the peritoneal cavity. D. Osmoreceptors in the hypothalamus stimulate thirst, which causes the stimulation to take in fluids orally. E. Overactivity of the enlarged spleen results in increased removal of blood cells from the circulation, which decreases the vascular pressure.

A. There is decreased colloid oncotic pressure from the liver's inability to synthesize albumin. B. Hyperaldosteronism related to damaged hepatocytes increases sodium and fluid retention. C. Portal hypertension pushes proteins from the blood vessels, causing leaking into the peritoneal cavity. The ascites related to cirrhosis are caused by decreased colloid oncotic pressure from the lack of albumin from liver inability to synthesize it and the portal hypertension that shifts the protein from the blood vessels to the peritoneal cavity, and hyperaldosteronism which increases sodium and fluid retention. The intake of fluids orally and the removal of blood cells by the spleen do not directly contribute to ascites

If a patient experiences chronic symptoms of cholelithiasis and cholecystitis, the recommended nutrition therapy is a. a low-fat diet. b. gradual weight loss. c. increased fluid intake. d. a low-cholesterol diet.

ANS: A A low-fat diet is used to treat painful symptoms associated with cholelithiasis and cholecystitis. Gradual weight loss may be beneficial in the long term, but will not decrease painful symptoms. Increased fluid intake and a low-cholesterol diet do not alleviate symptoms.

A symptom that is common to all types of hepatitis is a. jaundice. b. headache. c. dehydration. d. muscle aches.

ANS: A All types of hepatitis cause jaundice. Hepatitis E causes flu-like aches and pains, including headache. Dehydration may occur if patients have nausea and vomiting.

Cholecystitis is caused by a. blockage of the bile duct by gallstones, bacterial infection, or ischemia. b. concentration of bile in the gallbladder that favors formation of gallstones. c. failure of the gallbladder to contract and release bile into the small intestine. d. intake of excessive amounts of cholesterol and fat combined with bacterial infection.

ANS: A Cholecystitis occurs when gallstones block the cystic duct or as the result of stasis, bacterial infection, or ischemia of the gallbladder. Concentration of bile in the gallbladder causes cholelithiasis, or formation of gallstones. Failure of the gallbladder to contract and release bile may lead to gallstone formation; this may be caused by very low fat intake or dieting. Intake of excessive amounts of cholesterol and fat and bacterial infection are not associated with gallbladder disease.

3. A type of hepatitis that is transmitted via the fecal-oral route is hepatitis a. A. b. B. c. C. d. D.

ANS: A Hepatitis A is transmitted via the fecal-oral route. Hepatitis B and D are transmitted parenterally or sexually. Hepatitis C is transmitted via blood or serum (sharing of contaminated needles, razors, toothbrushes, nail files, barber's scissors, tattooing equipment, body piercing, or acupuncture needles).

An individual may be at risk for hepatitis E if they travel to India and eat a. curried shrimp. b. fresh fruit salad. c. Tandoori chicken. d. cooked foods from street vendors.

ANS: B Hepatitis E is transmitted via the fecal-oral route; food prepared by infected food handlers may transmit the disease. Raw fruits and vegetables (e.g., fruit salad) are common sources of infection. Foods that are cooked, such as curried shrimp and Tandoori chicken, and foods prepared by street vendors, are not common sources.

After surgical removal of the gallbladder (cholecystectomy), long-term dietary recommendations are a. a low-fat, low-cholesterol diet. b. high protein and fluid intakes. c. a well-balanced diet with no other restrictions. d. small, frequent meals to ensure adequate intake.

ANS: C After cholecystectomy, long-term dietary restrictions are not needed. Some patients need to restrict fat intake for a few weeks during recovery, but not long term. High protein and fluid intakes and small, frequent meals are not needed.

A patient with end-stage liver disease may lose fat stores and muscle mass, but this may not be evident from measurements of body weight because of a. dehydration. b. fat redistribution. c. ascites and edema. d. electrolyte imbalances.

ANS: C Patients with end-stage liver disease often accumulate fluid due to ascites and edema. This increases body weight, which may mask fat and muscle losses. Patients with end-stage liver disease are not usually dehydrated and do not usually have electrolyte imbalances. Fat infiltrates the liver but is not otherwise redistributed in the body.

For patients with hepatitis, a significant barrier to maintaining an adequate intake of kcals is a. malabsorption. b. fat intolerance. c. loss of appetite. d. increased metabolic rate.

ANS: C Patients with hepatitis often have very little appetite, which makes it hard for them to achieve adequate oral intake of nutrients. Patients with hepatitis do not usually have problems with malabsorption, fat intolerance, or increased metabolic rate.

The recommended diet for patients with hepatitis is a well-balanced diet with a. low protein content. b. supplemental electrolytes. c. no alcoholic beverages. d. limited amounts of alcohol.

ANS: C Total abstinence from alcohol is imperative for patients with hepatitis. The diet should be high in protein and kcals; supplemental electrolytes are not generally needed.

The patient with right upper quadrant abdominal pain has an abdominal ultrasound that reveals cholelithiasis. What should the nurse expect to do for this patient? A. Prevent all oral intake. B. Control abdominal pain. C. Provide enteral feedings. D. Avoid dietary cholesterol.

Ans: B Patients with cholelithiasis can have severe pain, so controlling pain is important until the problem can be treated. NPO status may be needed if the patient will have surgery but will not be used for all patients with cholelithiasis. Enteral feedings should not be needed, and avoiding dietary cholesterol is not used to treat cholelithiasis.

3. The family of a patient newly diagnosed with hepatitis A asks the nurse what they can do to prevent becoming ill themselves. Which response by the nurse is mostappropriate? A. "The hepatitis vaccine will provide immunity from this exposure and future exposures." B. "I am afraid there is nothing you can do since the patient was infectious before admission." C. "You will need to be tested first to make sure you don't have the virus before we can treat you." D. "An injection of immunoglobulin will need to be given to prevent or minimize the effects from this exposure."

Ans: D Immunoglobulin provides temporary (1-2 months) passive immunity and is effective for preventing hepatitis A if given within 2 weeks after exposure. It may not prevent infection in all persons, but it will at least modify the illness to a subclinical infection. The hepatitis vaccine is only used for preexposure prophylaxis

A patient who has hepatitis B surface antigen (HBsAg) in the serum is being discharged with pain medication after knee surgery. Which medication order should the nurse question because it is most likely to cause hepatic complications? A. Tramadol (Ultram) B. Hydromorphone (Dilaudid) C. Oxycodone with aspirin (Percodan) D. Hydrocodone with acetaminophen (Vicodin)

Ans: D The analgesic with acetaminophen should be questioned because this patient is a chronic carrier of hepatitis B and is likely to have impaired liver function. Acetaminophen is not suitable for this patient because it is converted to a toxic metabolite in the liver after absorption, increasing the risk of hepatocellular damage.

When teaching the patient with acute hepatitis C (HCV), the patient demonstrates understanding when the patient makes which statement? A. "I will use care when kissing my wife to prevent giving it to her." B. "I will need to take adofevir (Hepsera) to prevent chronic HCV." C. "Now that I have had HCV, I will have immunity and not get it again." D. "I will need to be checked for chronic HCV and other liver problems."

Ans: D The majority of patients who acquire HCV usually develop chronic infection, which may lead to cirrhosis or liver cancer. HCV is not transmitted via saliva, but percutaneously and via high-risk sexual activity exposure. The treatment for acute viral hepatitis focuses on resting the body and adequate nutrition for liver regeneration. Adofevir (Hepsera) is taken for severe hepatitis B (HBV) with liver failure. Chronic HCV is treated with pegylated interferon with ribavirin. Immunity with HCV does not occur as it does with HAV and HBV, so the patient may be reinfected with another type of HCV.

The nurse is reviewing the record of a female client with Crohn's disease. Which stool characteristics should the nurse expect to note documented in the client's record 1. Diarrhea 2. Chronic constipation 3. Constipation alternating with diarrhea 4. Stools constantly oozing form the rectum

Answer 1: Diarrhea, Crohn's disease is characterized by nonbloody diarrhea and around 4-5 stools per day. Over time, episodes of diarrhea increase in frequency, duration, and severity.

"A nurse is caring for a client admitted to the hospital with a suspected diagnosis of acute appendicitis. Which of the following laboratory results would the nurse expect to note if the client does have appendicitis? 1. Leukopenia with a shift to the right 2. Leukocytosis with a shift to the right 3.Leukocytosis with a shift to the left 4. Leukopenia with a shift to the left"

Answer 2

"In planning care for the patient with crohns disease the nurse recognizes that a major difference between UC and Crohn's disease is that: 1. Frequently results in toxic megacolon 2. causes fewer nutritional deficiencies than does UC 3. Often recurs after surgery whereas UC is curable with a colectomy 4. is manifested by rectal bleeding and anemia more frequently than UC"

Answer 3, "Medication is the primary treatment for Crohn's disease"

The client is diagnosed with an acute exacerbation of IBD. Which priority intervention should the nurse implement first? 1. Weigh the client daily and document it in the client's chart. 2. Teach coping strategies such as dietary modifications. 3. Record the frequency, amount, and color of stools. 4. Monitor the client's oral fluid intake every shift."

Answer 3, Record the frequency, amount, and color of stools"Rationale by answer option: 1. Weighing the client daily will help identify if the client is experiencing malnutrition, but it is not the priority intervention during an acute exacerbation. 2. Coping strategies help develop healthy ways to deal with this chronic disease that has remissions and exacerbations, but it is not the priority intervention. 3. The severity of the diarrhea helps determine the need for fluid replacement. The liquid stool should be measured as part of the total output. (CORRECT) 4. The client will be NPO when there is an acute exacerbation of IBD to allow the bowel to rest."

"Which associated disorder might a client with ulcerative colitis exhibit " 1. Gallstone 2. Hyronephrosis 3.Nephrolithiasis 4. Toxic megacolon

Answer 3, Toxic megacolon is extreme dilation of a segemnt of the diseased colon caused by paralysis of the colon

A patient returns to his room following a lower GI series. When he is assessed by the nurse, he complains of weakness. Which of the following nursing diagnoses should receive priority in planning his care? 1. Alteration in sensation - gustatory 2. Constipation, colonic 3. High risk for fluid volume deficit 4. Nutrition, less than body requirements

Answer 3, high risk for fluid vol. deficitprep for test: low-residue or clear liquid diet 2 days, NPO midnight, enemas, laxatives, post-test: laxatives to remove barium

"Which sign/symptom should the nurse expect to find in a client diagnosed with ulcerative colitis? 1. Twenty bloody stools a day. 2.Oral temperature of 102 ̊F. 3. Hard, rigid abdomen. 4. Urinary stress incontinence."

Answer = 1. The colon is ulcerated and unable to absorb water, resulting in bloody diar- rhea. Ten (10) to 20 bloody diarrhea stools is the most common symptom of ulcerative colitis. 2. Inflammation usually causes an elevated temperature but is not expected in the client with ulcerative colitis. 3.A hard, rigid abdomen indicates peritonitis, which is a complication of ulcerative colitis but not an expected symptom. 4. Stress incontinence is not a symptom of colitis."

"The nurse is caring for a patient in the emergency department with complaints of acute abdominal pain, nausea, and vomiting. When the nurse palpates the patient's left lower abdominal quadrant, the patient complains of pain in the right lower quadrant. The nurse will document this as which of the following diagnostic signs of appendicitis? "a. Rovsing sign b. referred pain c. Chvostek's sign d. rebound tenderness correct answer: A"

Answer A In patients with suspected appendicitis, Rovsing sign may be elicited by palpation of the left lower quadrant, causing pain to be felt in the right lower quadrant.

The nurse is teaching a female client how to perform a colostomy irrigation. To enhance the effectiveness of the irrigation and fecal returns, what measure would the nurse tell the client to do? "a. Increase fluid intake b. Place heat on the abdomen c. Perform the irrigation in the evening d. Reduce the amount of irrigation solution"

Answer A. To enhance effectiveness of the irrigation and fecal returns, the client is instructed to increase fluid intake and to take other measures to prevent constipation. Options B, C and D will not enhance the effectiveness of this procedure.

What is one of the major precipitating factors in the development of irritable bowel syndrome (IBS)? "A. Stress B. Peptic ulcers C. GERD D. Helicobacter pylori"

Answer A: Stress, Rationale: Stress is one of the major factors for developing irritable bowel syndrome (IBS), along with dietary factors.

A client is admitted to the hospital with viral hepatitis, complaining of "no appetite" and "losing my taste for food." What instruction should the nurse give the client to provide adequate nutrition? "a. Select foods high in fat b. Increase intake of fluids, including juices. c. Eat a good supper when anorexia is not as severe. d. Eat less often, preferably only three large meals daily."

Answer B : Although no special diet is required to treat viral hepatitis, it is generally recommended that clients consume a low-fat diet because fat may be tolerated poorly because of decreased bile production. Small frequent meals are preferable and may even prevent nausea. Frequently, appetite is better in the morining, so it is easier to eat a good breakfast. An adequated fluid intake of 2500 to 3000 mL/day that includes nutritional juices is also important.

"When preparing a male client, age 51, for surgery to treat appendicitis, the nurse formulates a nursing diagnosis of Risk for infection related to inflammation, perforation, and surgery. What is the rationale for choosing this nursing diagnosis? "a. Obstruction of the appendix may increase venous drainage and cause the appendix to rupture. b. Obstruction of the appendix reduces arterial flow, leading to ischemia, inflammation, and rupture of the appendix. c. The appendix may develop gangrene and rupture, especially in a middle-aged client. d. Infection of the appendix diminishes necrotic arterial blood flow and increases venous drainage."

Answer B. A client with appendicitis is at risk for infection related to inflammation, perforation, and surgery because obstruction of the appendix causes mucus fluid to build up, increasing pressure in the appendix and compressing venous outflow drainage. The pressure continues to rise with venous obstruction; arterial blood flow then decreases, leading to ischemia from lack of perfusion. Inflammation and bacterial growth follow, and swelling continues to raise pressure within the appendix, resulting in gangrene and rupture. Geriatric, not middle-aged, clients are especially susceptible to appendix rupture.

The nurse is caring for a male client postoperatively following creation of a colostomy. Which nursing diagnosis should the nurse include in the plan of care? -a. sexual dysfunction b. body image, disturbed c. fear related to poor prognosis d. Nutrition: more than body requirements, imbalanced

Answer B. Body image, disturbed relates to loss of bowel control, the presence of a stoma, the release of fecal material onto the abdomen, the passage of flatus, odor, and the need for an appliance (external pouch). No data in the question support options A and C. Nutrition: less than body requirements, imbalanced is the more likely nursing diagnosis.

THe nurse is caring for a client with a diagnosis of Crohn's disease. When evaluating a clients response to healthcare intervention, which expected outcome is the most important for the client: A. does skincare B. takes oral fluids C. gains .5 lb per week D. experiences less abdominal cramping

Answer C = gains .5lbs/wk, weight loss usually is severe with Crohn's disease, therefore, weight gain is a priority. this goal is specific, realistic, measureable and has a timeframe.

In planning care for the patient with Crohn's disease, the nurse recognizes that a major difference between ulcerative colitis and Crohn's disease is that Crohn's disease A. frequently results in toxic megacolon, B. causes fewer nutritional deficiencies than does ulcerative colitis, C. often recurs after surgery, whereas ulcerative colitis is curable with a colectomy, D. is manifested by rectal bleeding and anemia more frequently than is ulcerative colitis.

Answer C, often recurs after surgery, whereas ulcerative colitis is curable with a colectomyRationale: Because there is a high recurrence rate after surgical treatment of Crohn's disease, medications are the preferred treatment.

The nurse is caring for a hospitalized female client with a diagnosis of ulcerative colitis. Which finding, if noted on assessment of the client, would the nurse report to the physician? "a. Hypotension b. Bloody diarrhea c. Rebound tenderness d. A hemoglobin level of 12 mg/dL"

Answer C. Rebound tenderness may indicate peritonitis. Bloody diarrhea is expected to occur in ulcerative colitis. Because of the blood loss, the client may be hypotensive and the hemoglobin level may be lower than normal. Signs of peritonitis must be reported to the physician.

During the assessment of a patient with acute abdominal pain, the nurse should: a. perform deep palpation before ausculation b. obtain BP and pulse rateto determine hypervolemic changes c. auscultate bowel sounds because hyperactive bowel sounds suggest paralytic ileus d. measure body temperature because and elevated temperature may indicate an inflammatory or infectious process.

Answer D, If the temperature is elevated pain may be due to infection.

A nurse is caring for a child who had a laproscopic appendectomy. What interventions should the nurse document on the child's clinical record? Select all that apply. 1) Intake and Output 2) Measurement of Pain 3) Tolerance to low-residue diet 4) Frequency of dressing changes 5) Auscultation of bowel sounds

Answer: 1, 2, 5 1) Assessment and documentation of fluid balance are critical aspects of all postoperative care. 2) Laparoscopic surgery involves insufflating the abdominal cavity with air, which is painful until it is absorbed. The amount of pain should be measured and documented with either a 1-10 scale or the Wong's FACES for younger children. 3) A special diet is not indicated after this surgery. 4) After a laparoscopic appendectomy there is little drainage and no dressings. 5) Auscultating for bowel sounds and documenting their presennce or absence evaluate the child's adaptation to the intestinal trauma caused by the surgery.

The nurse is assessing an adolescent who is admitted to the hospital with appendicitis. The nurse should report which of the following to the HCP? "1) change in pain rating of 7 to 8 on a 10 point scale. 2) sudden relief of sharp pain, shifting to diffuse pain. 3)shallow breathing with normal vital signs. 4) decrease of pain rating from 8 to 6 when parents visit.

Answer: 2 Rationale: The nurse notifies the HCP if the client has sudden relief of sharp pain and on presence of more diffuse pain. this change in the pain indicates the appendix has ruprured. The diffuse pain is typically accompanied by rigid guarding of the abdomen, progressive abdominal distension, tachycardia, pallor, chills, and irritability. The slight increase pain can be expected; the decrease in pain when parents visit may be attributed to being distracted from the pain. shallow breathing is likely due to the pain and is insignificant when other vital signs are normal

Which of the nursing interventions should be implemented to manage appendicitis? a. Assess pain b. encourage oral intake of clear fluids. c. provide discharge teaching D. assess for symptoms of peritonitis. answer D. Monitor for peritonitis because if the appendix ruptures, bacteria can enter the peritoneum. Pain will be managed with analgesics, and pt should be NPO for surgery. Discharge is not done at this time A client with complaints of right lower quadrant pain is admitted to the emergency department. Blood specimens are drawn and sent to the laboratory. Which laboratory finding should be reported to the physician immediately? "a) Hematocrit 42% b) Serum potassium 4.2 mEq/L c) Serum sodium 135 mEq/L d) White blood cell (WBC) count 22.8/mm3.

Answer: D "D) White blood cell (WBC) count 22.8/mm3 The nurse should report the elevated WBC count. This finding, which is a sign of infection, indicates that the client's appendix might have ruptured. Hematocrit of 42%, serum potassium of 4.2 mEq/L, and serum sodium of 135 mEq/L are within normal limits. Alterations in these levels don't indicate appendicitis."

Bobby, a 13 year old is being seen in the emergency room for possible appendicitis. An important nursing action to perform when preparing Bobby for an appendectomy is to:""a) administer saline enemas to cleanse the bowels b) apply heat to reduce pain c) measure abdominal girth d) continuously monitor pain

Answer: D Rationale: Pain is closely monitored in appendicitis. In most cases, pain medication is not given until prior to surgery or until the diagnosis is confirmed to be able to closely monitor the progression of the disease. A sudden change in the character of pain may indicate rupture or bowel perforation. Administering an enema or applying heat may cause perforation and abdominal girth may not change with appendicitis.

the nurse plans the care for a client with hepatitis A with the understanding that the causative virus will be excreted from the client's body primarily through the: A. skin B. Feces C. Urine D. Blood

B

A patient is admitted with an abrupt onset of jaundice, nausea and vomiting, hepatomegaly, and abnormal liver function studies. Serologic testing is negative for viral causes of hepatitis. Which question by the nurse is most appropriate? a. "Have you been around anyone with jaundice?" b. "Do you use any prescription or over-the-counter (OTC) drugs?" c. "Are you taking corticosteroids for any reason?" d. "Is there any history of IV drug use?"

B Rationale: The patient's symptoms, lack of antibodies for hepatitis, and the ABRUPT onset of symptoms suggest toxic hepatitis, which can be caused by commonly used OTC drugs such as acetaminophen (Tylenol). Exposure to a jaundiced individual and a history of IV drug use are risk factors for VIRAL hepatitis. Corticosteroid use does not cause the symptoms listed.

The nurse determines that teaching regarding cobalamin injections has been effective when the patient with chronic atrophic gastritis states which of the following? a. "The cobalamin injections will prevent gastric inflammation." b. "The cobalamin injections will prevent me from becoming anemic." c. "These injections will increase the hydrochloric acid in my stomach." d. "These injections will decrease my risk for developing stomach cancer

B Cobalamin supplementation prevents the development of pernicious anemia. Chronic gastritis may cause achlorhydria, but cobalamin does not correct this. The loss of intrinsic factor secretion with chronic gastritis is permanent, and the patient will need lifelong supplementation with cobalamin. The incidence of stomach cancer is higher in patients with chronic gastritis, but cobalamin does not reduce the risk for stomach cancer.

Which information will the nurse include for a patient with newly diagnosed gastroesophageal reflux disease (GERD)? a. "Peppermint tea may reduce your symptoms." b. "Keep the head of your bed elevated on blocks." c. "You should avoid eating between meals to reduce acid secretion." d. "Vigorous physical activities may increase the incidence of reflux."

B Elevating the head of the bed will reduce the incidence of reflux while the patient is sleeping. Peppermint will decrease lower esophageal sphincter (LES) pressure and increase the chance for reflux. Small, frequent meals are recommended to avoid abdominal distention. There is no need to make changes in physical activities because of GERD.

Which nursing action should be included in the postoperative plan of care for a patient after a laparoscopic esophagectomy? a. Notify the doctor about bloody nasogastric (NG) drainage. b. Elevate the head of the bed to at least 30 degrees. c. Reposition the NG tube if drainage stops. d. Start oral fluids when the patient has active bowel sounds.

B Elevation of the head of the bed decreases the risk for reflux and aspiration of gastric secretions. The NG tube should not be repositioned without consulting with the health care provider. Bloody NG drainage is expected for the first 8 to 12 hours. A swallowing study is needed before oral fluids are started.

A 26-year-old patient with a family history of stomach cancer asks the nurse about ways to decrease the risk for developing stomach cancer. The nurse will teach the patient to avoid a. emotionally stressful situations. b. smoked foods such as ham and bacon. c. foods that cause distention or bloating. d.

B Smoked foods such as bacon, ham, and smoked sausage increase the risk for stomach cancer. Stressful situations, abdominal distention, and use of H2 blockers are not associated with an increased incidence of stomach cancer.

A patient is admitted to the hospital with acute cholecystitis. Which assessment information will be most important for the nurse to report to the health care provider? a. The patient's urine is bright yellow. b. The patient's stools are clay colored. c. The patient complains of chronic heartburn. d. The patient has an increase in pain after eating.

B The clay-colored stools indicate biliary obstruction, which requires rapid intervention to resolve. The other data are not unusual for a patient with this diagnosis, although the nurse also would report the other assessment information to the health care provide

the nurse is planning a community education program on how to prevent the transmission of viral hepatitis. which of the following types of hepatitis is considered to primarily a sexually transmitted disease. A. Hep A B. Hep B C. Hep C D. Hep D

B. Hep B is considered to be sexually transmitted. Hep C is less frequently transmitted sexually and more frequently percutaneously

which of the following measures would prevent transmission of the hepatitis C virus to health care personnel A. administering hepatitis C vaccine to all health care personnel B. decreasing contact with blood and blood-contaminated fluids C. wearing gloves when emptying the bedpan D. wearing a gown and mask when providing direct care

B. hep C is usually transmitted through blood exposure and needlesticks

the nurse should expect the client to exhibit which of the following S/S during icteric phase of viral hepatitis. A. tarry stools B. yellowed sclera C. SOB D. light, frothy urine

B. Excess bilirubin turns skin and sclera yellow and urine dark and frothy

The patient with right upper quadrant abdominal pain has an abdominal ultrasound that reveals cholelithiasis. What should the nurse expect to do for this patient? A. Prevent all oral intake. B. Control abdominal pain. C. Provide enteral feedings. D. Avoid dietary cholesterol.

B. Control abdominal pain. Patients with cholelithiasis can have severe pain, so controlling pain is important until the problem can be treated. NPO status may be needed if the patient will have surgery but will not be used for all patients with cholelithiasis. Enteral feedings should not be needed, and avoiding dietary cholesterol is not used to treat cholelithiasis

Which of the following definitions best describes diverticulosis? A. An inflamed outpouching of the intestine B. A noninflamed outpouching of the intestine C. The partial impairment of the forward flow of intestinal contents D. An abnormal protrusion of an organ through the structure that usually holds it.

B. Diverticulosis involves a noninflamed outpouching of the intestine. Diverticulitis involves an inflamed outpouching. The partial impairment of forward flow of the intestine is an obstruction; abnormal protrusion of an organ is a hernia.

A nurse is admitting a patient with the diagnosis of advanced renal carcinoma. Based upon this diagnosis, the nurse will expect to find what clinical manifestations as the "classic triad" occurring in patients with renal cancer? A. Fever, chills, flank pain B. Hematuria, flank pain, palpable mass C. Hematuria, proteinuria, palpable mass D. Flank pain, palpable abdominal mass, and proteinuria

B. Hematuria, flank pain, palpable mass There are no characteristic early symptoms of renal carcinoma. The classic manifestations of gross hematuria, flank pain, and a palpable mass are those of advanced disease.

A 68-year-old male patient with a stroke is unconscious and unresponsive to stimuli. After learning that the patient has a history of gastroesophageal reflux disease (GERD), the nurse will plan to do frequent assessments of the patient's a. apical pulse. b. bowel sounds. c. breath sounds. d. abdominal girth.

C Because GERD may cause aspiration, the unconscious patient is at risk for developing aspiration pneumonia. Bowel sounds, abdominal girth, and apical pulse will not be affected by the patient's stroke or GERD and do not require more frequent monitoring than the routine.

A 46-year-old female with gastroesophageal reflux disease (GERD) is experiencing increasing discomfort. Which patient statement indicates that additional teaching about GERD is needed? a. "I take antacids between meals and at bedtime each night." b. "I sleep with the head of the bed elevated on 4-inch blocks." c. "I eat small meals during the day and have a bedtime snack." d. "I quit smoking several years ago, but I still chew a lot of gum."

C GERD is exacerbated by eating late at night, and the nurse should plan to teach the patient to avoid eating at bedtime. The other patient actions are appropriate to control symptoms of GERD.

Which patient choice for a snack 2 hours before bedtime indicates that the nurse's teaching about gastroesophageal reflux disease (GERD) has been effective? a. Chocolate pudding b. Glass of low-fat milk c. Cherry gelatin with fruit d. Peanut butter and jelly sandwich

C Gelatin and fruit are low fat and will not decrease lower esophageal sphincter (LES) pressure. Foods such as chocolate are avoided because they lower LES pressure. Milk products increase gastric acid secretion. High-fat foods such as peanut butter decrease both gastric emptying and LES pressure.

an 18 yr old is admitted with an acute onset of right lower quadrant pain. Appendicitis is suspected. For which clinical indicator should the nurse assess the client to determine if the pain is secondary to appendicitis A) urinary retention B) gastric hyperacidity C) rebound tenderness D) increased lower bowel motility

C) rebound tenderness is a classic subjective sign of appendicitis

Following bowel resection, a patient has a nasogastric tube to suction, but complains of nausea and abdominal distention. The nurse irrigates the tube prn as ordered, but the irrigating fluid does not return. Which of the following should be the priority action by the nurse? A. Notify the physician B. Auscultate for bowel sounds. C. Reposition the tube and check for placement. D. Remove the tube and replace it with a new one.

C, reposition the tube and check for placementThe tube may be resting against the stomach wall. The first action by the nurse, since this was intestinal surgery (not gastric surgery), is to reposition the tube and check it again for placement.

which of the following test results should the nurse use to assess the liver function of a client with viral hepatitis A. glucose tolerance B. creatinine clearance C. serum transaminase D. serum electrolytes

C. along with bilirubin and other enzymes transaminase provide important data about liver function

which of the following diets would most likely be prescribed for a client with viral hepatitis A. high-fat, low-protein B. high-protein, low-carb C. high-carb, high-calorie D. low-sodium, low-fat

C. because these clients are generally anorexic and have little interest in eating

the develops a teaching plan for the client about how to prevent the transmission of hepatitis A. which of the following discharge instructions is appropriate for the client A. spray the house to eliminate infected insects B. tell the family to stay away from the client C. tell the family to wash their hands frequently D. disinfect all clothing and eating utensils

C. hep A is transmitted through fecal oral route. hand washing is essential

interferon alfa-2b (intron A) has been prescribed to treat a client with chronic hepatitis B. what adverse effect is most commonly associated with the administration of interferon alfa-2b A. retinopathy B. constipation C. flulike symptoms D. hypoglycemia

C. such as myalia, arthralgia, headache, nausea, fever, and fatigue

A patient with cholelithiasis needs to have the gallbladder removed. Which patient assessment is a contraindication for a cholecystectomy? A. Low-grade fever of 100° F and dehydration B. Abscess in the right upper quadrant of the abdomen C. Activated partial thromboplastin time (aPTT) of 54 seconds D. Multiple obstructions in the cystic and common bile duct

C. Activated partial thromboplastin time (aPTT) of 54 seconds An aPTT of 54 seconds is above normal and indicates insufficient clotting ability. If the patient had surgery, significant bleeding complications postoperatively are very likely. Fluids can be given to eliminate the dehydration; the abscess can be assessed, and the obstructions in the cystic and common bile duct would be relieved with the cholecystectomy.

An older male patient visits his primary care provider because of burning on urination and production of urine that he describes as "foul smelling." The health care provider should assess the patient for what factor that may put him at risk for a urinary tract infection (UTI)? A. High-purine diet B. Sedentary lifestyle C. Benign prostatic hyperplasia (BPH) D. Recent use of broad-spectrum antibiotics

C. Benign prostatic hyperplasia (BPH) BPH causes urinary stasis, which is a predisposing factor for UTIs. A sedentary lifestyle and recent antibiotic use are unlikely to contribute to UTIs, whereas a diet high in purines is associated with renal calculi.

Which of the following definitions best describes gastritis? A. Erosion of the gastric mucosa B. Inflammation of a diverticulum C. Inflammation of the gastric mucosa D. Reflux of stomach acid into the esophagus

C. Gastritis is an inflammation of the gastric mucosa that may be acute (often resulting from exposure to local irritants) or chronic (associated with autoimmune infections or atrophic disorders of the stomach). Erosion of the mucosa results in ulceration. Inflammation of a diverticulum is called diverticulitis; reflux of stomach acid is known as gastroesophageal disease.

Nurse is caring for a patient with a diagnosis of ulcerative colitis. Which finding, if noted on assessment of the client, would the nurse report to the Dr? A. Hypotension B. Bloody diarrhea C. Rebound tenderness D. Hemoglobin of 12 mg/ dl

C. Rebound tenderness because this could indicate peritonitis

A client has surgery for a perforated appendix with localized peritonis. In which position should the nurse place the client? A) Sims position B) trendelenburg C) semi-fowlers D)dorsal recumbant

C. Semi-fowlers aids in drainage and prevents spread of infection throughout the abodominal cavity.

A client is admitted to the hospital with a diagnosis of cholecystitis from cholelithiasis. The client has severe abdominal pain and nausea, and has vomited several times. Based on these data, which nursing action would have been the highest priority for intervention at this time? a. Manage anxiety b. Restore fluid loss c. Manage the pain d. Replace nutritional loss

C: The priority for nursing care at this time is to decrease the client's severe abdominal pain. The pain, which is frequently accompanied by nausea and vomiting, is caused by biliary spasm.

A client with Crohn's disease is admitted to the hospital with a history of chronic, bloody diarrhea, weight loss, and signs of general malnutrition. The client has anemia, a low serum albumin level, and signs of negative nitrogen balance. The nurse concludes that the client's health status is related to a major deficiency of: 1. Iron 2. Protein 3. Vitamin C 4. Linoleic acid

CORRECT ANSWER 2: Protein deficiency causes a low serum albumin level, which permits fluid shifts from the intravascular to the interstitial compartment, resulting in edema. Decreased protein also causes anemia; protein intake must be increased. Although a deficiency of iron will result in anemia, it will not cause the other adaptations. Vitamin C and linoleic acid are unrelated to these adaptations.

The nurse is monitoring a client diagnosed with appendicitis who is scheduled for surgery in 2 hours. The client begins to complain of increased abdominal pain and begins to vomit. On assessment, the nurse notes that the abdomen is distended and bowel sounds are diminished. Which is the appropriate nursing intervention? " 1. Notify the Physician 2. Administer the prescribed pain medication 3. Call and ask the operating room team to perform the surgery as soon as possible 4. Reposition the client and apply a heating pad on warm setting to the client's abdomen

CORRECT ANSWER: 1" "1. Based on the assessment information the nurse should suspect peritonitis, a complication that is associated with appendicitis, and notify the physician. 2. Administering pain medication is not an appropriate intervention 3. Scheduling surgical time is not within the scope of practice of an RN. 4. Heat should never be applied to the abdomen of a patient suspected of having peritonitis because of the risk of rupture."

The nurse is planning to teach a client with GERD about substances to avoid. Which items should the nurse include on this list? Select all that apply. Coffee Chocolate Peppermint Nonfat milk Fried chicken Scrambled eggs

Coffee, chocolate, peppermint, fried chicken

"A client is admitted with right lower quadrant pain, anorexia, nausea, low-grade fever, and an elevated white blood cell count. Which complication is most likely the cause? "1. A fecalith 2. Bowel kinking 3. Internal bowel occlusion 4. Abdominal wall swelling"

Correct 1 The client is experiencing appendicitis. A fecalith is a fecal calculus, or stone, that occludes the lumen of the appendix and is the most common cause of appendicitis. Bowel wall swelling, kinking of the appendix, and external occlusion, not internal occlusion, of the bowel by adhesions can also be causes of appendicitis.

A client complains of severe pain in the right lower quadrant of the abdomen. To assist with pain relief, the nurse should take which of the following actions? "1. Encourage the client to change positions frequently in bed 2. Massage the right lower quadrant fo the abdomen 3. Apply warmth to the abdomen with a heating pad 4. Use comfort measures and pillows to position the client"

Correct 4 "1. ""Encourage the client..."" - unnecesary movement will increase pain and should be avoided 2. ""Massage the lower..."" - if appendicitis is suspected, massage or palpation should never be performed as these actions may cause the appendix to rupture 3. ""Apply warmth..."" - if pain is caused by appendicitis, increased circulation from the heat may cause appendix to rupture 4. ""Use comfort measures..."" - CORRECT: non-pharmacological methods of pain relief"

"A client with acute appendicitis develops a fever, tachycardia, and hypotension. Based on these assessment findings, the nurse should further assess the client for which of the following complications?... "1. Deficient fluid volume. 2. Intestinal obstruction. 3. Bowel ischemia. 4. Peritonitis

Correct 4 "Complications of acute appendicitis are perforation, peritonitis, and abscess development. Signs of the development of peritonitis include abdominal pain and distention, tachycardia, tachypnea, nausea, vomiting, and fever. Because peritonitis can cause hypovolemic shock, hypotension can develop. Deficient fluid volume would not cause a fever. Intestinal obstruction would cause abdominal distention, diminished or absent bowel sounds, and abdominal pain. Bowel ischemia has signs and symptoms similar to those found with intestinal obstruction."

"A nurse is providing wound care to a client 1 day after the client underwent an appendectomy. A drain was inserted into the incisional site during surgery. Which action should the nurse perform when providing wound care? 1. Remove the dressing and leave the incision open to air. 2. Remove the drain if wound drainage is minimal. 3. Gently irrigate the drain to remove exudate. 4. Clean the area around the drain moving away from the drain.

Correct 4 The nurse should gently clean the area around the drain by moving in a circular motion away from the drain. Doing so prevents the introduction of microorganisms to the wound and drain site. The incision cannot be left open to air as long as the drain is intact. The nurse should note the amount and character of wound drainage, but the surgeon will determine when the drain should be removed. Surgical wound drains are not irrigated.

When assessing a patient who is scheduled to have a CT scan of the kidneys, which of these findings would prompt the nurse to notify the primary healthcare provider? 1. allergy to iodine and seafood 2. . urinary output of 1,200 mL in 24 hours 3. last bowel movement one day ago 4. height 5'8" and weight 160 pounds

Correct Answer: 1 Rationale: Allergy to iodine and seafood is correct because a CT scan of the kidneys requires the injection of a radiopaque dye that contains iodine. A patient who is allergic to iodine or seafood will be unable to have this test. Urinary output of 1,200 mL in 24 hours, last bowel movement one day ago, and height 5'8" and weight 160 pounds are all incorrect because these are all normal findings, and therefore do not require that the physician be notified.

A nurse is assessing a 68-year-old female patient who states, "I am having episodes of urinary incontinence." The nurse should recognize this statement as indicating which of the following? 1. an abnormal finding requiring further testing 2. an indication of the presence of a urinary infection 3. a normal outcome of the aging process 4. the result of having several children

Correct Answer: 1 Rationale: An abnormal finding requiring further testing is correct because incontinence is not a normal part of the aging process, and therefore will require further investigation to identify the cause. An indication of the presence of a urinary infection is incorrect because although frequency and urgency can be symptoms of a urinary tract infection, a culture and sensitivity test is necessary in order to determine infection. A normal outcome of the aging process and a result of having several children are incorrect because incontinence is not normal, and is it not necessarily the result of having had several children.

A nursing student is assessing a patient who is reporting constant dull pain over the lower abdomen. The student inspects, palpates, and auscultates the patient's abdomen. After leaving the patient's room the nurse tells the student, "Your assessment findings may not be accurate because you 1. palpated prior to auscultating." 2. inspected prior to palpating." 3. inspected prior to auscultating." 4. auscultated after inspecting."

Correct Answer: 1 Rationale: Auscultate immediately after inspection because percussion or palpation may increase bowel motility and interfere with sound transmission during auscultation.

A nurse is caring for a patient who asks the nurse why females are more likely than males to contract bladder infections. The nurse knows teaching has been effective when the patient identifies which of the following as a female risk factor for bladder infections? 1. The urinary meatus is closer to the bladder than in most males. 2. The urinary meatus is farther from the anus than most males. 3. The pH of the female urethra is more conducive to infection. 4. Females urinate more frequently than males, increasing risk.

Correct Answer: 1 Rationale: In females, the urethra is approximately 1.5 inches (3 to 5 cm) long, and the urinary meatus is anterior to the vaginal orifice. In males, the urethra is approximately 8 inches (20 cm) long. The shorter distance of the female urethra creates a mechanism by which more females than males contract bladder infections. The female urinary meatus is closer, not farther from the anus than in most males, also increasing risk for bladder infections. The pH of the female urethra is not more conducive to infection. Frequent urination decreases the risk of bladder infection making this choice incorrect.

A nurse is assessing a patient. Which of the following patient statements best alerts the nurse to the likelihood of the patient having a distended bladder? 1. "I am in pain and it is worse when I press on my abdomen." 2. "My back is killing me." 3. "It feels like someone is stabbing me in the abdomen with a knife." 4. "It hurt constantly with spasms once in a while."

Correct Answer: 1 Rationale: The patient with a distended bladder experiences constant pain increased by any pressure over the bladder. Kidney pain is experienced in the back and the costovertebral angle (the angle between the lower ribs and adjacent vertebrae) and may spread toward the umbilicus. Renal colic (pain in response to renal calculi moving through the ureter) is severe, sharp, stabbing, and excruciating; often it is felt in the flank, bladder, urethra, testes, or ovaries. Bladder and urethral pain is usually dull and continuous but may be experienced as spasms.

The nurse is caring for a patient who sustained a fall with a fractured femur and was unable to summon help or receive healthcare treatment for 48 hours. On arrival at the emergency department, the patient's blood urea nitrogen level is 50 mg/dL. The serum creatinine level is 1.0 mg/dL. These findings would help substantiate a nursing diagnosis of which of the following? 1. Deficient Fluid Volume 2. Anxiety related to crisis 3. Acute Pain 4. Impaired Nutrition

Correct Answer: 1 Rationale: To assess if the patient's elevated blood urea nitrogen is caused by dehydration or renal failure, the nurse assesses the serum creatinine value. The patient's serum creatinine is normal, which does not indicate kidney failure. A nursing diagnosis of Deficient Fluid Volume is appropriate for this patient.

The nurse assesses a patient admitted to the medical-surgical unit who has a diagnosis of type I diabetes mellitus. The nurse notes that the patient's urine is cloudy and foul-smelling. Which of the following diagnostic tests does the nurse anticipate will be ordered based on this finding? 1. urine culture and sensitivity (C&S) 2. blood urea nitrogen (BUN) 3. creatinine clearance 4. residual urine

Correct Answer: 1 Rationale: Urine culture and sensitivity (C&S) is correct because cloudy and foul-smelling urine indicates a urinary tract infection. The diagnostic test to identify the organism responsible is a urine C&S. Blood urea nitrogen (BUN) measures the amount of urea (end product of protein metabolism) in the blood plasma. It does not identify infection. Creatinine clearance is a 24-hour urine test used to identify renal function; it will not identify an infection. Residual urine measures the amount of urine left in the bladder after voiding, and does not identify an infection.

Which of the following position should the client with appendicitis assume to relieve pain ? A. Prone B. Sitting C. Supine D. Lying with legs drawn up

Correct Answer: D Lying still with legs drawn up towards chest helps relive tension on the abdominal muscle, which helps to reduce the amount of discomfort felt. Lying flat or sitting may increase the amount of pain experienced

A nurse is making a home health visit and finds the client experiencing right lower quadrant abdominal pain, which has decreased in intensity over the last day. The client also has a rigid abdomen and a temperature of 103.6 F. The nurse should intervene by: a) administer Tylenol (acetaminophen) for the elevated temperature b) advising the client to increase oral fluids c) asking the client when she last had a bowel movement d) notifying the physician

Correct D D. The client symptoms indicate appendicitis which requires immediate attention

Which of the following would indicate that Bobby's appendix has ruptured? " a) diaphoresis b) anorexia c) pain at Mc Burney's point d) relief from pain

Correct D all are normal signs of having appendicits and once you have relief from pain means you could have a rupture.

A client with cirrhosis of the liver develops ascites. Which of the following orders would the nurse expect? A. Restrict fluid to 1000 mL per day. B. Ambulate 100 ft. three times per day. C. High-sodium diet. D. Maalox 30 ml P.O. BID.

Correct answer: A Fluid restriction is a primary treatment for ascites. Restricting fluids decreases the amount of fluid present in the body, thereby decreasing the fluid that accumulates in the peritoneal space. A high sodium diet would increase fluid retention. Physical activities are usually restricted until ascites is relieved. Loop diuretics (such as furosemide) are usually ordered, and Maalox® (a bismuth subsalicylate) may interfere with the action of the diuretics.

The nurse is doing teaching with the family of a client with liver failure. Which of the following foods should the nurse advise them to limit in the client's diet? A. Meats and beans. B. Butter and gravies. C. Potatoes and pasta. D. Cakes and pastries.

Correct answer: A Meats and beans are high-protein foods and are restricted with liver failure. In liver failure, the liver is unable to metabolize protein adequately, causing protein by-products to build up in the body rather than be excreted. This causes problems such as hepatic encephalopathy (neurologic syndrome that develops as a result of rising blood ammonia levels). Although other nutrients, such as fat and carbohydrates, may be regulated, it's most important to limit protein in the diet of the client with liver failure.

A patient with hepatitis A is in the acute phase. The nurse plans care for the patient based on the knowledge that a. pruritus is a common problem with jaundice in this phase. b. the patient is most likely to transmit the disease during this phase. c. gastrointestinal symptoms are not as severe in hepatitis A as they are in hepatitis B. d. extrahepatic manifestations of glomerulonephritis and polyarteritis are common in this phase.

Correct answer: a Rationale: The acute phase of jaundice may be icteric or anicteric. Jaundice results when bilirubin diffuses into the tissues. Pruritus sometimes accompanies jaundice. Pruritus is the result of an accumulation of bile salts beneath the skin.

A patient with acute hepatitis B is being discharged in 2 days. In the discharge teaching plan the nurse should include instructions to a. avoid alcohol for the first 3 weeks. b. use a condom during sexual intercourse. c. have family members get an injection of immunoglobulin. d. follow a low-protein, moderate-carbohydrate, moderate-fat diet

Correct answer: b Rationale: Hepatitis B virus may be transmitted by mucosal exposure to infected blood, blood products, or other body fluids (e.g., semen, vaginal secretions, saliva). Hepatitis B is a sexually transmitted disease that is acquired through unprotected sex with an infected person. Condom use should be taught to patients to prevent transmission of hepatitis B.

"A client is admitted with a diagnosis of acute appendicitis. When assessing the abdomen, the nurse would expect to find rebound tenderness at which location? a) Left lower quadrant b) Left upper quadrant c) Right upper quadrant d) Right lower quadrant

Correct answer: d) Right lower quadrant" Rationale: The pain of acute appendicitis localizes in the right lower quadrant (RLQ) at McBurney's point, an area midway between the umbilicus and the right iliac crest. Often, the pain is worse when manual pressure near the region is suddenly released, a condition called rebound tenderness.

"The nurse is admitting a client with acute appendicitis to the emergency department. The client has abdominal pain of 10 on a pain scale of 1 to 10. The client will be going to surgery as soon as possible. The nurse should: "1. Contact the surgeon to request an order for a narcotic for the pain. 2. Maintain the client in a recumbent position. 3. Place the client on nothing-by-mouth (NPO) status. 4. Apply heat to the abdomen in the area of the pain."

Correct: 3

"A client has an appendectomy and develops peritonitis. The nurse should asses the client for an elevated temperature and which additional clinical indication commonly associated with peritonitis? "1. hyperactivity 2. extreme hunger 3. urinary retention 4. local muscular rigidity

Correct: 4 muscular rigidity over the affected area is a classic sign of peritonitis

"The health care team is assessing a patient for acute pancreatitis after he presented to the emergency department with severe abdominal pain. Which laboratory value is the best diagnostic indicator of acute pancreatitis? A. Gastric pH B. Blood glucose C. Serum amylase D. Serum potassium

Correct: C Serum amylase levels indicate pancreatic function, and they are used to diagnose acute pancreatitis. Blood glucose, gastric pH, and potassium levels are not direct indicators of acute pancreatic dysfunction.

Which client requires immediate nursing intervention? "The client who: a) complains of epigastric pain after eating. b) complains of anorexia and periumbilical pain. c) presents with ribbonlike stools. d) presents with a rigid, boardlike abdomen.

Correct: D A rigid, boardlike abdomen is a sign of peritonitis, a possibly life-threatening condition. Epigastric pain occurring 90 minutes to 3 hours after eating indicates a duodenal ulcer. Anorexia and periumbilical pain are characteristic of appendicitis. Risk of rupture is minimal within the first 24 hours, but increases significantly after 48 hours. A client with a large-bowel obstruction may have ribbonlike stools.

A health care provider who has not been immunized for hepatitis B is exposed to the hepatitis B virus (HBV) through a needle stick from an infected patient. The infection control nurse informs the individual that treatment for the exposure should include a. baseline hepatitis B antibody testing now and in 2 months. b. active immunization with hepatitis B vaccine. c. hepatitis B immune globulin (HBIG) injection. d. both the hepatitis B vaccine and HBIG injection.

D Rationale: The recommended treatment for exposure to hepatitis B in unvaccinated individuals is to receive both HBIG and the hepatitis B vaccine, which would provide temporary passive immunity and promote active immunity. Antibody testing may also be done, but this would not provide protection from the exposure.

The nurse will anticipate teaching a patient experiencing frequent heartburn about a. a barium swallow. b. radionuclide tests. c. endoscopy procedures. d. proton pump inhibitors.

D Because diagnostic testing for heartburn that is probably caused by gastroesophageal reflux disease (GERD) is expensive and uncomfortable, proton pump inhibitors are frequently used for a short period as the first step in the diagnosis of GERD. The other tests may be used but are not usually the first step in diagnosis.

When caring for a patient following an incisional cholecystectomy for cholelithiasis, the nurse places the highest priority on assisting the patient to a. choose low-fat foods from the menu. b. perform leg exercises hourly while awake. c. ambulate the evening of the operative day. d. turn, cough, and deep breathe every 2 hours.

D Postoperative nursing care after a cholecystectomy focuses on prevention of respiratory complications because the surgical incision is high in the abdomen and impairs coughing and deep breathing. The other nursing actions also are important to implement but are not as high a priority as ensuring adequate ventilation.

Which assessment should the nurse perform first for a patient who just vomited bright red blood? a. Measuring the quantity of emesis b. Palpating the abdomen for distention c. Auscultating the chest for breath sounds d. Taking the blood pressure (BP) and pulse

D The nurse is concerned about blood loss and possible hypovolemic shock in a patient with acute gastrointestinal (GI) bleeding. BP and pulse are the best indicators of these complications. The other information is important to obtain, but BP and pulse rate are the best indicators for assessing intravascular volume

The nurse explaining esomeprazole (Nexium) to a patient with recurring heartburn describes that the medication a. reduces gastroesophageal reflux by increasing the rate of gastric emptying. b. neutralizes stomach acid and provides relief of symptoms in a few minutes. c. coats and protects the lining of the stomach and esophagus from gastric acid. d. treats gastroesophageal reflux disease by decreasing stomach acid production.

D The proton pump inhibitors decrease the rate of gastric acid secretion. Promotility drugs such as metoclopramide (Reglan) increase the rate of gastric emptying. Cryoprotective medications such as sucralfate (Carafate) protect the stomach. Antacids neutralize stomach acid and work rapidly.

A 50-year-old man vomiting blood-streaked fluid is admitted to the hospital with acute gastritis. To determine possible risk factors for gastritis, the nurse will ask the patient about a. the amount of saturated fat in the diet. b. any family history of gastric or colon cancer. c. a history of a large recent weight gain or loss. d. use of nonsteroidal antiinflammatory drugs (NSAIDs).

D Use of an NSAID is associated with damage to the gastric mucosa, which can result in acute gastritis. Family history, recent weight gain or loss, and fatty foods are not risk factors for acute gastritis.

A client with cholelithiasis has a gallstone lodged in the common bile duct. When assessing this client, the nurse expects to note: a) black, tarry stools. b) circumoral pallor. c) light amber urine. d) yellow sclerae.

D) Yellow sclerae Yellow sclerae are an early sign of jaundice, which occurs when the common bile duct is obstructed. Urine normally is light amber. Circumoral pallor and black, tarry stools don't occur in common bile duct obstruction; they are signs of hypoxia and GI bleeding, respectively.

A client with acute ulcerative colitis requests a snack. Which of the following foods is the most appropriate to give the client? A. Carrots and ranch dip B. Whole grain cereal and milk C. A cup of popcorn and a cola D. Applesauce and a graham cracker

D, appel sauce and graham cracker, The diet for a client with ulcerative coliits should be a low-fiber, low residue diet. The nurse should avoid foods such as whole grains, nuts and fresh fruit or vegetables. Typically lactose containing foods are also poorly tolerated. The client should also avoid caffeine, pepper, and alcohol.

The nurse is caring for patients in the student health center. A patient confides to the nurse that the patient's boyfriend informed her that he tested positive for Hepatitis B. Which of the following responses by the nurse is BEST? a. "That must have been a real shock to you" b. "You should be tested for Hepatitis B" c. "You'll receive the Hepatitis B immune globulin HBIG d. "Have you had unprotected sex with your boyfriend"

D. Hepatitis B is transmitted through parenteral drug abuse and sexual contact. Determine exposure before implementing.

a client who is recovering from hepatitis A continues to complain of fatigue and malaise. the client asks the nurse, "when will my strength return?" which of the following responses is most appropriate A. your fatigue should be gone by now. we will evaluate you for a secondary infection B. you fatigue is an adverse effect of your drug therapy. it will disappear once your regimen is complete C. it is important for you to increase your activity level. that will help you decrease fatigue D. it is normal for you to feel fatigue. the fatigue show go away in the next 2-4 months

D. during the convalescent or posticteric stage, fatigue and malaise are common complaints

the nurse is preparing a community education program about preventing hepatitis B infection. which of the following would be appropriate to incorporate into the teaching plan A. hepaitis B is relatively uncommon among college students B. frequent ingestion of alcohol can predispose a client to hepatitis B C. good personal hygiene habits are most effective at preventing the spread of hepatits B D. the use of a condom is advised for sexual intercourse

D. hepatitis B is considered a sexual transmitted disease

The nurse is giving general instructions to a client receiving hemodialysis. Which statement would be most appropriate for the nurse to include? A. "It is acceptable to eat whatever you want on the day before hemodialysis." B. "It is acceptable to exceed the fluid restriction on the day before hemodialysis." C. "Medications should be double-dosed on the morning of hemodialysis because of potential loss." D. "Several types of medications should be withheld on the day of dialysis until after the procedure."

D. "Several types of medications should be withheld on the day of dialysis until after the procedure." Many medications are dialyzable, which means that they are extracted from the bloodstream during dialysis. Therefore many medications may be withheld on the day of dialysis until after the procedure. It is not typical for medications to be double-dosed, because there is no way to be certain how much of each medication is cleared by dialysis. Clients receiving hemodialysis are not routinely taught that it is acceptable to disregard dietary and fluid restrictions.

Colon cancer is most closely associated with which of the following conditions? A. Appendicitis B. Hemorrhoids C. Hiatal hernia D. Ulcerative colitis

D. Chronic ulcerative colitis, granulomas, and familial polposis seem to increase a person's chance of developing colon cancer. The other conditions listed have no known effect on colon cancer risk.

Which of the following mechanisms can facilitate the development of diverticulosis into diverticulitis? A. Treating constipation with chronic laxative use, leading to dependence on laxatives B. Chronic constipation causing an obstruction, reducing forward flow of intestinal contents C. Herniation of the intestinal mucosa, rupturing the wall of the intestine D. Undigested food blocking the diverticulum, predisposing the area to bacteria invasion.

D. Undigested food can block the diverticulum, decreasing blood supply to the area and predisposing the area to invasion of bacteria. Chronic laxative use is a common problem in elderly clients, but it doesn't cause diverticulitis. Chronic constipation can cause an obstruction—not diverticulitis. Herniation of the intestinal mucosa causes an intestinal perforation.

After a cholecystectomy, the client is to follow a low-fat diet. Which of the following foods would be most appropriate to include a low-fat diet? a. Cheese omlet b. Peanut butter c. Ham salad sandwich d.Roast Beef

D: Lean meats, such as beef, lamb, veal, and well trimmed lean ham and pork, are low in fat. Cheese omlet, peanut butter, and ham salad are high in fat.

A client has undergone a laparoscopic cholecystectomy. Which of the following instructions should the nurse include in the discharge teaching? a. Empty the bile bag daily b. Breathe deeply into a paper bag when nauseated c. Keep adhesive dressings in place for 6 weeks d. Report bile-colored drainage from any incision

D: There should be no bile colored drainage coming from any of the incisions postoperatively. A laparoscopic cholecystectomy does not involve a bile bag. Breathing into a paper bag will prevent a person from passing out due to hyperventilation; it does not alleviate nausea. If the adhesives have not already fallen off, they are removed by the surgeon in 7-10 days, not 6 weeks.

In planning care for a patient with ulcerative colitis, the nurse should anticipate which of the following diagnostic procedures? a. sigmoidodscopy b. colonoscopy, c. rectal mucosa biopsy, d. all of the above

Diagnosis of ulcerative colitis is confirmed with the use of sigmoidoscopy, colonoscopy, and rectal mucosa biopsy.

"Older patients with longstanding or severe Crohn's disease can exhibit which of the following? a Hyperalbuminemia b)Hypoalbuminemia c) Decreased sedimentation rate d)Nausea and vomiting

Hypoalbuminemia, Rationale wasn't given for this question but Crohns affects the GI tract which will affect the ability to absorb protein so it is lost through the urine.

A client who has a history of Crohn's disease is admitted to the hospital with fever, diarrhea, cramping, abdominal pain, and weight loss. The nurse should monitor the client for 1. Hyperalbuminemia. 2. Thrombocytopenia. 3. Hypokalemia. 4. Hypercalcemia.

Hypokalemia is the most expected laboratory finding owing to the diarrhea. Hypoalbuminemia can also occur in Crohn's disease; however, the client's potassium level is of greater importance at this time because a low potassium level can cause cardiac arrest. Anemia is an expected development, but thrombocytopenia is not. Calcium levels are not affected.

A 74 y.o. female pt w/ GERD takes over-the-counter meds. For which med, if taken long-term, should the nurse teach about increased risk of fractures? Sucralfate (Carafate) Cimetidine (Tagamet) Omeprazole (Prilosec) Metoclopramide (Reglan)

Omeprazole (Prilosec) There is a potential link between proton pump inhibitor use and bone metabolism. Long-term use or high doese of these may increase the risk of fractures of the hip, wrist, and spine. Lower doses or shorter duration of therapy should be considered

"The client diagnosed with appendicitis has undergone an appendectomy. At two hours postoperative, the nurse takes the vital signs and notes T 102.6 F, P 132, R 26, and BP 92/46. Which interventions should the nurse implement? List in order of priority. 1. Increase the IV rate. 2. Notify the health care provider. 3. Elevate the foot of the bed. 4. Check the abdominal dressing. 5. Determine if the IV antibiotics have been administered.

Order of priority: 1, 3, 4, 5, 2." "1. The nurse should increase the IV rate to maintain the circulatory system function until further orders can be obtained. 3. The foot of the bed should be elevated to help treat shock, the symptoms of which include elevated pulse and decreased BP. Those signs and an elevated temperature indicate an infection may be present and the client could be developing septicemia. 4. The dressing should be assessed to determine if bleeding is occurring. 5. The nurse should administer any IV antibiotics ordered after addressing hypovolemia. The nurse will need this information when reporting to the HCP. 2. The HCP should be notified when the nurse has the needed information."

A nurse is caring for a patient who has undergone a knee joint replacement. What measures should a nurse take to prevent constipation of the patient in the postoperative period? Select all that apply. Correct 1 Advise the patient to drink more than 2500mL/day of fluids. 2 Instruct the patient to drink cold fluids. Correct 3 Advise the patient to eat more fruits and vegetables. 4 Advise the patient to maintain complete bed rest until recovery. Correct 5 Use stool softeners and laxatives as advised.

Patients often have reduced mobility after a fracture, which may result in constipation. The nurse should implement appropriate measures, such as high fluid intake (more than 2500mL/day unless contraindicated) and a diet high in bulk and roughage (fruits and vegetables) to prevent constipation. If these measures fail to maintain normal bowel pattern, then laxatives and stool softeners can be used. Constipation can be relieved by drinking warm fluids, not cold ones. Physical activity also helps in bowel activity, so the patient should ambulate as early as the indications and provider prescriptions allow.

"The most common surgical procedures for patients with ulcerative colitis are: 1.Subtotal colectomy and ileostomy. 2.Colostomy and ileo-conduit. 3.Laparoscopic gastrectomy. 4.Segmental resection or

Rationale: Surgery might be necessary for functional older patients with acute disease when drug therapy fails and when multiple precancerous lesions are detected. The most common surgical procedures are subtotal colectomy and ileostomy.

A patient is admitted to the orthopedic surgical unit for a fracture of the left tibia. What instructions should a nurse give concerning the prevention of venous thromboembolism? Select all that apply. Correct 1 Wear compression gradient stockings. 2 Perform range-of-motion exercises on the left lower limb. Correct 3 Perform range-of-motion exercises on the right lower limb. Correct 4 Exercise toes of the left lower limb against resistance. 5 Exercise toes of the right lower limb against resistance.

There is a high risk of venous thromboembolism in the orthopedic surgical patient. Therefore measures should be taken by the attending nurse to prevent it. These measures include instructing the patient to wear compression gradient stockings (antiembolism hose) and to use sequential compression devices. The patient should also be encouraged to move (dorsiflex and plantar flex) the fingers or toes of the affected extremity against resistance and perform range-of-motion exercises on the unaffected lower extremities.

A patient with a fracture of the right tibia is scheduled for application of a cast. What is the correct order of applying the materials, from inside to outside? Incorrect 1.Immerse plaster of Paris in warm water. Incorrect 2.Cover the affected part with a stockinette. Incorrect 3.Place padding over the stockinette. Correct 4.Wrap plaster of Paris around the affected part.

To apply a cast on an extremity, first cover the affected part with a stockinette that is cut longer than the extremity. Then place padding over the stockinette, with the bony prominences given extra padding. If the casting material used is plaster of Paris, it is usually immersed in warm water before being wrapped and molded around the affected part. The number of layers of plaster bandage and the technique of application determine the strength of the cast. The plaster sets within 15 minutes.

Which diagnostic test is used first to evaluate a client with upper GI bleeding? "a) Hemoglobin levels and hematocrit (HCT) b) Endoscopy c) Arteriography d) Upper GI series

Which diagnostic test is used first to evaluate a client with upper GI bleeding? "a) Hemoglobin levels and hematocrit (HCT) b) Endoscopy c) Arteriography d) Upper GI series

The client hemodialyzed suddenly becomes short of breath and complains of chest pain. The client is tachycardic, pale and anxious. The nurse suspects air embolism. The priority action for the nurse is to: a) discontinue dialysis and notify the physician b) monitor vital signs every 15 minutes for the next hour c) continue dialysis at a slower rate after checking the lines for air d) bolus the client with 500 ml of normal saline to break up the air embolus

a) discontinue dialysis and notify the physician

Which of the following should be considered in the diet of the client with end-stage-renal-disease (ESRD)? a) limit fluid intake during anuric phase b) limit phosphorus and vitamin D-rich food c) limit calcium-rich food d) limit carbohydrates

a) limit fluid intake during anuric phase during ESRD, fluid intake of the client should be limited during anuric phase to prevent fluid overload. Fluid overload increases renal workload, pulmonary edema, and congestive heart failure.

The client with continuous ambulatory peritoneal dialysis (CAPD) has cloudy dialysate. Which of the following is the best initial nursing action? a) send fluid to the laboratory for culture b) administer antibiotic c) do nothing, this is expected d) stop drainage of fluid

a) send fluid to the laboratory for culture cloudy diasylate indicates infection (peritonitis). Culture of the fluid must be done to determine the microorganism present.

The nurse performs a detailed assessment of the abdomen of a patient with a possible bowel obstruction, knowing that a manifestation of an obstruction in the large intestine is (select all that apply) a. a largely distended abdomen. b. Diarrhea that is loose or liquid. c. persistent, colicky abdominal pain. d. profuse vomiting that relieves abdominal pain.

a, c

Which of the following characterize acute kidney injury (select all that apply) a. primary cause of death is infection b. almost always affects older people c. disease course is potentially reversible d. most common cause is diabetic nepropathy e. cardiovascular disease is most common cause of death

a,c

What is the function of a tendon? a. Attaches muscle to bone b. Connects bone to bone at the joint c. Connects cartilage to muscle in joints d. Attaches synovium to the joint capsule

a. Attaches muscle to bone The function of the tendon is to attach muscle to bone. The ligament attaches bone to bone at the joint. Fascia encloses individual muscles but does not connect cartilage to muscle in joints. The bursae are lined with a synovial membrane and are located in joints. The bursae are lined with a synovial membrane and are located in joints to relieve pressure and decrease friction between moving parts.

After having a transverse colostomy constructed for colon cancer, discharge planning for home care would include teaching about the ostomy appliance. Information appropriate for this intervention would include: a. Instructing the client to report redness, swelling, fever, or pain at the site to the physician for evaluation of infection b. Nothing can be done about the concerns of odor with the appliance. c. Ordering appliances through the client's health care provider d. The appliance will not be needed when traveling.

a. Instructing the client to report redness, swelling, fever, or pain at the site to the physician for evaluation of infection Signs and symptoms for monitoring infection at the ostomy site are a priority evaluation for clients with new ostomies. The remaining actions are not appropriate. There are supplies avaliable for clients to help control odor that may be incurred because of the ostomy. Although a prescription for ostomy supplies is needed, you can order the supplies from any medical supplier. Dependent on the location and trainability of the ostomy, appliances are almost always worn throughout the day and when traveling

A client has just had surgery for colon cancer. Which of the following disorders might the client develop? a. Peritonitis b. Diverticulosis c. Partial bowel obstruction d. Complete bowel obstruction

a. Peritonitis Bowel spillage could occur during surgery, resulting in peritonitis. Complete or partial bowel obstruction may occur before bowel resection. Diverticulosis doesn't result from surgery or colon cancer.

Radiation therapy is used to treat colon cancer before surgery for which of the following reasons? a. Reducing the size of the tumor b. Eliminating the malignant cells c. Curing the cancer d. Helping the bowel heal after surgery

a. Reducing the size of the tumor Radiation therapy is used to treat colon cancer before surgery to reduce the size of the tumor, making it easier to be resected. Radiation therapy isn't curative, can't eliminate the malignant cells (though it helps define tumor margins), can could slow postoperative healing.

The nurse explains to the patient undergoing ostomy surgery that the procedure that maintains the most normal functioning of the bowel is a. a sigmoid colostomy b. a transverse colostomy c. a descending colostomy d. an ascending colostomy

a. a sigmoid colostomy

Which of the following diets is most commonly associated with colon cancer? a. low fiber, high fat b. low fat high fiber c. low protein, high carb d. low carb, high protein

a. a. low fiber, high fat low fiber, high fat diet reduced motility and increases the chance of constipation. The metabolic end products of this type of diet are carcinogenic. A LOW FAT HIGH FIBER diet is recommended to help avoid colon cancer. Carbohydrates and protein aren't necessarily associated with colon cancer.

A closed amputation is usually performed to a. create a weight bearing residual limb. b. alleviate the effects of the trauma c. allow infection to heal and drain d. treat a limb with gangrene

a. create a weight bearing residual limb

Preoperative exercises for a patient undergoing a lower extremity amputation include a. upper body training b. lower body training c. upper thigh training d. head and neck training

a. upper body training

The nurse is admitting a client with the diagnosis of appendicitis to the surgical unit. Which question is essential to ask? A."When did you last eat?" B."Have you had surgery before?" C."Have you ever had this type of pain before?" D."What do you usually take to relieve your pain?"

answer A. When a person is admitted with possible appendicitis, the nurse should anticipate surgery. It will be important to know when she last ate when considering the type of anesthesia so that the chance of aspiration can be minimized. The other inoformation is "nice to know", but not essential.

Which of the nursing interventions should be implemented to manage appendicitis? a. Assess pain b. encourage oral intake of clear fluids. c. provide discharge teaching D. assess for symptoms of peritonitis.

answer D. Monitor for peritonitis because if the appendix ruptures, bacteria can enter the peritoneum. Pain will be managed with analgesics, and pt should be NPO for surgery. Discharge is not done at this time

One of the major advantages of peritoneal dialysis is that a. no medications are required because of the enhanced efficiency b. the diet is less restricted and dialysis can be performed at home c. the dialysate is biocompatible and causes no long term consequences d. high glucose concentrations of the dialysate cause a reduction in appetite promoting weight loss

b

Teaching in relation to home management after a laparoscopic cholecystectomy should include a. keeping the bandages on the puncture sites for 48 hours. b. reporting any bile-colored drainage or pus from any incision. c. using over-the-counter antiemetics if nausea and vomiting occur. d. emptying and measuring the contents of the bile bag from the T tube every day. (Lewis 1042)

b Rationale: The following discharge instructions are taught to the patient and caregiver after a laparoscopic cholecystectomy: First, remove the bandages on the puncture site the day after surgery and shower. Second, notify the surgeon if any of the following signs and symptoms occur: redness, swelling, bile-colored drainage or pus from any incision; and severe abdominal pain, nausea, vomiting, fever, or chills. Third, gradually resume normal activities. Fourth, return to work within 1 week of surgery. Fifth, resume a usual diet, but a low-fat diet is usually better tolerated for several weeks after surgery.

During the oliguric phase of AKI, the nurse monitors the patient for (select all that apply) a. hypotension b. ECG changes c. hypernatremia d. pulmonary edema e. urine with high specific gravity

b,d

Which of the following symptoms is a client with colon cancer most likely to exhibit? a. A change in appetite b. A change in bowel habits c. An increase in body weight d. An increase in body temperature

b. A change in bowel habits The most common complaint of the client with colon cancer is a change in bowel habits. The client may have anorexia, secondary abdominal distention, or weight loss. Fever isn't associated with colon cancer.

While preparing the client for a colonoscopy, the nurse's responsibilities include: a. Explaining the risks and benefits of the exam b. Instructing the client about the bowel preparation prior to the test c, Instructing the client about medication that will be used to sedate the client d. Explaining the results of the exam

b. Instructing the client about the bowel preparation prior to the test The nurse is responsible for instructing the client about the bowel preparation prior to the test. Answers 1, 3, 4 are the physician's responsibility.

Cool clammy, mottled skin that is pale in the patient undergoing amputation is important to note in the assessment because it may indicate a. a poor oxygen supply to the involved tissues b. a poor blood supply to the involved tissues c. the tissue is edematous d. that an infection is present

b. a poor blood supply to the involved tissues

The nurse would increase the comfort of the patient with appendicitis by a. having the patient lie prone. b. flexing the patient's right knee. c. sitting the patient upright in a char. d. turn the patient onto his or her left side.

b. flexing the patient's right knee.

The greatest danger in the early postoperative period after an amputation is: a. infection b. hemorrhage c. pain d. edema

b. hemorrhage

In contrast to diverticulitis, the patient with diverticulosis a. has no rectal bleeding b. often has no symptoms c. has localized cramping pain d. frequently develops peritonitis

b. often has no symptoms

A patient returns to your unit with a postoperative infection after an above-knee amputation. The patient tells you that he is "going crazy" because he thinks he still feels his toes. You know the reason for this complication is: a. phantom limb pain b. phantom limb sensation c. pain d. contracture

b. phantom limb sensation

A patient with metastatic colorectal cancer is scheduled for both chemotherapy and radiation therapy. Patient teaching regarding these therapies for this patient would include an explanation that a. Chemotherapy can be used to cure colorectal cancer. b radiation is routinely used as adjuvant therapy following surgery. c. both chemotherapy and radiation can be used a palliative treatments. d. the patient should expect few if any side effects from chemotherapeutic agents.

c

A adult client has had laboratory work done as part of a routine physical examination. The nurse interprets that the client may have a mild degree of renal insufficiency if which of the following serum creatinine levels is noted? a) 0.2 mg/dlL b) 0.5 mg/dL c) 1.9 mg/dL d) 3.5 mg/dL

c) 1.9 mg/dL the normal serum creatinine level foadults is 0.6 to 1.3 mg/dL. The client with a mild degree of renal insufficiency would have a slight elevated level. A creatinie level of 0.2 mg/dL is low, and a level of 0.5 mg/dL is just below normal. A creeatinie level of 3.5 mg/dL may be associated with acute or chronic renal failure.

A nurse is assessing a client who is diagnosed with cystitis. Which assessment finding is inconsistent with the typical clinical manifestations noted in this disorder? a) hematuria b) low back pain c) urinary retention d) burning on urination

c) urinary retention

In planning care for the patient with Crohn's disease, the nurse recognizes that a major difference between ulcerative colitis and Crohn's is that Crohn's disease a. frequently results in toxic megacolon. b. causes fewer nutritional deficiencies than does ulcerative colitis. c. often recurs after surgery, whereas ulcerative colitis is curable with a colectomy. d. Is manifested by rectal bleeding and anemia more frequently than is ulcerative colitis.

c. often recurs after surgery, whereas ulcerative colitis is curable with a colectomy.

A patient with cholelithiasis needs to have the gallbladder removed. Which patient assessment is a contraindication for a cholecystectomy? Low-grade fever of 100° F and dehydration Abscess in the right upper quadrant of the abdomen Activated partial thromboplastin time (aPTT) of 54 seconds

ctivated partial thromboplastin time (aPTT) of 54 seconds Multiple obstructions in the cystic and common bile duct An aPTT of 54 seconds is above normal and indicates insufficient clotting ability. If the patient had surgery, significant bleeding complications postoperatively are very likely. Fluids can be given to eliminate the dehydration; the abscess can be assessed, and the obstructions in the cystic and common bile duct would be relieved with the cholecystectomy.

RIFLE defines three stages of AKI based on changes in a. blood pressure and urine osmolality b. fractional excretion of urinary sodium c. estimation of GFR with the MDRD equation d. serum creatinine or urine output from baseline

d

The nursing management of the patient with cholecystitis associated with cholelithiasis is based on the knowledge that a. shock-wave therapy should be tried initially. b. once gallstones are removed, they tend not to recur. c. the disorder can be successfully treated with oral bile salts that dissolve gallstones. d. laparoscopic cholecystectomy is the treatment of choice in most patients who are symptomatic

d Rationale: Laparoscopic cholecystectomy is the treatment of choice for symptomatic cholelithiasis.

A client has an arteriovenous (AV) fistula in place in the right upper extremity for hemodialysis treatments. When planning care for this client, which of the following measures should the nurse implement to promote client safely? a) take blood pressures only on the right arm to ensure accuracy b) use the fistula for all venipunctures and intravenous infusions c) ensure that small clamps are attached to the AV fistula dressing d) assess the fistula for the presence of a bruit and thrill every 4 hours

d) assess the fistula for the presence of a bruit and thrill every 4 hours

Which of the following diagnostic tests should be performed annually over age 50 to screen for colon cancer? a. Abdominal CT scan b. Abdominal x-ray c. Colonoscopy d. Fecal occult blood test

d. Fecal occult blood test Surface blood vessels of polyps and cancers are fragile and often bleed with the passage of stools. Abdominal x-ray and CT scan can help establish tumor size and metastasis. A colonoscopy can help locate a tumor as well as polyps, which can be removed before they become malignant.

During the assessment of a patient with acute abdominal pain, the nurse should a. perform deep palpation before auscultation. b. obtain blood pressure and pulse rate to determine hypervolemic changes.. c. auscultate bowel sounds because hyperactive bowel sounds suggest paralytic ileus. d. Measure body temperature because an elevated temperature may indicate an inflammatory or infectious process.

d. Measure body temperature because an elevated temperature may indicate an inflammatory or infectious process.

Colon cancer is most closely associated with which of the following conditions? a. appendicitis b. hemorroids c. hiatal hernia d. ulcerative colitis

d. ulcerative colitis Chronic ulcerative colitis, granulomas, and familial polyps seem to increase a person's chance of developing colon cancer. The other conditions listed have no known effect on the colon cancer risk.


Set pelajaran terkait

Ways of examining the human body

View Set

MIS 513 - Midterm (Quizzes), MIS Midterm Chapter 1, Quiz 1, TRUE MIS 3302 UHD Exam 2 (Ch.5-8) Review, UHD M.I.S. 3302 Final (ALL CHAPTERS), MIS FINAL EXAM, IS Chapter 3, IS Chapter 4, MIS 304 4

View Set

Federal Fair Housing Laws & Protected Classes

View Set

Chapter 49: Disorders of Musculoskeletal Function: Developmental and Metabolic Disorders, Activity Intolerance, and Fatigue

View Set

Chapter 13 Central Nervous System

View Set